Крок 1 - Медицина 2021 перескладання (буклет)

1 / 158
В ході гістологічного дослідження біоптату, отриманого з нижньої третини стравоходу 57-річного чоловіка з симптомами тривалого рефлюксу шлункового вмісту, виявлено наступні зміни: у слизовій оболонці на місці багатошарового плаского епітелію спостерігається одношаровий залозистий призматичний епітелій, з ознаками продукції слизу. Вкажіть патологічний процес, який виник у слизовій оболонці: In the course of a histological examination of a biopsy obtained from the lower third of the esophagus of a 57-year-old man with symptoms of long-term reflux of gastric contents, the following changes were found: in the mucous membrane at the place of a multilayer flat single-layer glandular prismatic epithelium is observed in the epithelium, with signs of mucus production. Specify the pathological process that occurred in the mucous membrane:

Метаплазія Metaplasia

Гіпертрофія Hypertrophy

Регенерація Regeneration

Гіперплазія Hyperplasia

Організація Organization

2 / 158
У хворого після резекції шлунка з приводу виразкової хвороби перистальтика кишківника не відновилася. Який лікарський засіб доцільно призначити хворому для відновлення моторики? Intestinal motility did not return to the patient after gastric resection due to peptic ulcer disease. What medicine should be prescribed to the patient to restore motility?

Атенолол Atenolol

Резерпін Reserpin

Гігроній Hygronium

Метилурацил Methyluracil

Прозерин Proserin

3 / 158
В сироватці крові при постановці імуноферментної реакції був визначений НВ8-антиген. При якому захворюванні зустрічається даний антиген? HB8 antigen was detected in the blood serum during the immunoenzymatic reaction. In which disease is this antigen found?

СНІД AIDS

Туберкульоз Tuberculosis

Сифіліс Syphilis

Вірусний гепатит А Viral hepatitis A

Вірусний гепатит В Viral hepatitis B

4 / 158
У хворого на крупозну пневмонію має місце гарячка з температурою тіла 39°С, при цьому добові коливання температури не перевищували 1°С впродовж 9-ти діб. До якого типу температурних кривих відноситься ця гарячка? A patient with croup pneumonia has a fever with a body temperature of 39°C, while daily temperature fluctuations did not exceed 1°C for 9 days. Until which what type of temperature curves does this fever belong to?

Гіперпіретична Hyperpyretic

Поворотна Rotary

Гектична Hectic

Ремітуюча Remitting

Постійна Constant

5 / 158
У хворого 67-ми років з клінічним діагнозом хронічного бронхіту, пневмосклерозу, серцево-легеневої недостатності взято біоптат з підозрілої ділянки слизової оболонки правого бронха. Гістологічно встановлено клітинний і тканинний атипізм, появу структур у вигляді 'ракових перлин’.’ Якому патологічному процесу відповідають зазначені гістологічні зміни? A 67-year-old patient with a clinical diagnosis of chronic bronchitis, pneumosclerosis, cardiopulmonary failure had a biopsy taken from a suspicious area of the mucous membrane of the right bronchus. Cell and tissue were histologically determined atypism, the appearance of structures in the form of 'cancer pearls'. What pathological process do these histological changes correspond to?

Гострий бронхіт Acute bronchitis

Бронхоектаз Bronchiectasis

Хронічний поліпозний бронхіт Chronic polyposis bronchitis

Плоскоклітинна метаплазія слизової бронху Squamous metaplasia of the bronchial mucosa

Плоскоклітинний рак бронху з ороговінням Squamous cell carcinoma of the bronchus with keratinization

6 / 158
Після нападу бронхіальної астми хворому проведено дослідження периферичної крові. Які зміни очікуються? After an attack of bronchial asthma, the patient underwent a peripheral blood test. What changes are expected?

Лімфоцитоз Lymphocytosis

Лейкопенія Leukopenia

Еозинофілія Eosinophilia

Еритроцитоз Erythrocytosis

Тромбоцитопенія Thrombocytopenia

7 / 158
Людина знаходиться в середовищі з темепературою 38 градусів С, відносною вологістю повітря 50%. Які шляхи тепловіддачі зумовлюють підтримку постійної температури ядра тіла за цих умов? A person is in an environment with a temperature of 38 degrees C and a relative humidity of 50%. What heat transfer pathways ensure the maintenance of a constant body core temperature under these conditions?

Конвекція Convection

Теплопроведення Heat conduction

Конвекція і теплопроведення Convection and heat conduction

Випаровування Evaporation

Радіація Radiation

8 / 158
В клітинах здорової печінки активно синтезуються глікоген ТА білки. Які типи органел добре розвинуті? Glycogen AND proteins are actively synthesized in the cells of a healthy liver. What types of organelles are well developed?

Клітинний центр Cell Center

Лізосоми Lysosomes

Гранулярна та агранулярна ЕПС Granular and agranular EPS

Мітохондрії Mitochondria

Пероксисоми Peroxisomes

9 / 158
У людини з четвертою групою крові (генотип ІАІВ) в еритроцитах одночасно присутні антиген А, який контролюється алелем ІА, і антиген В - продукт експресії алеля ІВ. Прикладом якої взаємодії генів є дане явище? In a person with the fourth blood group (genotype IAIV), antigen A, which is controlled by the allele IA, and antigen B - a product of the expression of the allele IV, are simultaneously present in erythrocytes. An example of which is the interaction of genes a given phenomenon?

Неповного домінування Incomplete dominance

Кодомінування Codominance

Епістазу Epistasis

Полімерії Polymeria

Комплементарності Complementarities

10 / 158
До лікаря звернувся хворий 30-ти років з імовірною на ВІЛ-інфекцію. Для уточнення діагнозу лікар запропонував провести полімеразну ланцюгову реакцію. Основним процесом в даному дослідженні є: A 30-year-old patient with suspected HIV infection approached the doctor. To clarify the diagnosis, the doctor suggested conducting a polymerase chain reaction. The main process in this study is:

Транскрипція Transcription

Ампліфікація генів Gene Amplification

Хромосомна мутація Chromosome mutation

Генетична рекомбінація Genetic recombination

Генна мутація Gene mutation

11 / 158
Після травми у хворого спостерігається порушення моторних центрів, що регулюють діяльність м’язів голови. В яких ділянках кори півкуль в нормі локалізується відповідний центр? After an injury, the patient has a violation of the motor centers that regulate the activity of the head muscles. In which areas of the cortex of the hemispheres is the corresponding center normally located?

Верхня тім'яна часточка Superior parietal lobe

Нижня частина передцентральної звивини Inferior part of the precentral gyrus

Кутова звивина Angular gyrus

Верхня частина передцентральної звивини Upper part of precentral gyrus

Надкрайова звивина Supermarginal gyrus

12 / 158
Під час спортивних змагань боксер отримав сильний удар у живіт, що привело до нокауту через короткочасне падіння артеріального тиску. Які фізіологічні механізми викликали цей стан? During a sports competition, a boxer received a strong blow to the stomach, which led to a knockout due to a short-term drop in blood pressure. What physiological mechanisms caused this condition?

Ішемія ЦНС CNS ischemia

Підсилення парасимпатичних впливів Strengthening of parasympathetic influences

Раптова зміна кількості рідини у організмі Sudden change in the amount of fluid in the body

Подразнення симпатичних нервів Irritation of sympathetic nerves

Зміна транскапілярного обміну Transcapillary exchange change

13 / 158
Хворий висловлює скарги на біль у верхньому відділі пупкової ділянки. Пальпаторно визначається рухлива болісна кишка. Яку кишку пальпує лікар? The patient complains of pain in the upper part of the umbilical region. A mobile painful intestine is detected by palpation. Which intestine does the doctor palpate?

Дванадцятипала Decimal

Поперечно-ободова Transverse rim

Клубова Club

Сигмоподібна Sigma

Порожня Empty

14 / 158
У людини при переході зі світлого приміщення в темне відбувається розширення зіниць. Який з наведених рефлексів зумовлює цю реакцію? When a person moves from a bright room to a dark one, the pupils dilate. Which of the following reflexes causes this reaction?

Симпатичний умовний Cute Conditional

Метасимпатичний Metasympathetic

Парасимпатичний умовний Parasympathetic conditional

Симпатичний безумовний Lovely Unconditional

Парасимпатичний безумовний Parasympathetic unconditional

15 / 158
Чоловіку 35-ти років з виразковою хворобою зроблено резекцію антрального відділу шлунку. Секреція якого гастроінтестинального гормону внаслідок операції буде порушена найбільше? A 35-year-old man with peptic ulcer underwent resection of the antral part of the stomach. The secretion of which gastrointestinal hormone will be most affected as a result of the operation?

Гістамін Histamine

Нейротензин Neurotensin

Секретин Secretin

Холецистокінін Cholecystokinin

Гастрин Gastrin

16 / 158
У хворого на виразкову хворобу шлунка під час мікроскопії мазка-відбитка з біоптату слизової виявлені грамнегативні напівзігнуті дугоподібні мікроби, тест на уреазну активність позитивний. Які мікроби були виявлені у хворого? In a patient with gastric ulcer, during the microscopy of a smear-imprint from a mucosa biopsy, gram-negative semi-bent arcuate microbes were detected, the test for urease activity was positive. What microbes were detected in the patient ?

Спірохети Spirochetes

Лептоспіри Leptospira

Хелікобактерії Helicobacter pylori

Трепонеми Treponemes

Спірили Spiril

17 / 158
Для лікування ревматоїдного артриту жінці 65-ти років у складі комплексної терапії був призначений гормональний препарат, який має імунодепресивну дію. Визначте цей препарат: For the treatment of rheumatoid arthritis, a 65-year-old woman was prescribed a hormonal drug that has an immunosuppressive effect as part of complex therapy. Identify this drug:

Рибофлавін Riboflavin

Тималін Tymalin

Преднізолон Prednisone

Супрастин Suprastin

Ферковен Ferkoven

18 / 158
У чоловіка 41-го року відзначаються періодичні напади серцебиття (пароксизми), сильне потовиділення, напади головного болю. При обстеженні виявлена гіпертензія, гіперглікемія, підвищення основного обміну, тахікардія. При якій патології наднирників спостерігається подібна картина? A 41-year-old man has periodic attacks of heart palpitations (paroxysms), profuse sweating, and attacks of headache. During the examination, hypertension, hyperglycemia, increased basic metabolism, tachycardia were found In what pathology of the adrenal glands is a similar picture observed?

Гіпофункція кори наднирників Hypofunction of the adrenal cortex

Гіперфункція мозкового шару Hyperfunction of the cerebral layer

Гіперфункція кори наднирників Hyperfunction of the adrenal cortex

Первинний альдостеронізм Primary aldosteronism

Гіпофункція мозкового шару Hypofunction of the cerebral layer

19 / 158
На гістологічному препараті нирки в дистальному звивистому канальці виявляються клітини, які щільно прилягають до ниркового тільця. Базальна мембрана їх дуже тонка і не утворює складок. Ці клітини відчувають зміни вмісту натрію у сечі та впливають на секрецію реніну юкстагломерулярними клітинами. Які це клітини? On the histological preparation of the kidney, cells are found in the distal convoluted tubule that are closely adjacent to the renal corpuscle. Their basement membrane is very thin and does not form folds. These cells experience changes in the content sodium in the urine and affect renin secretion by juxtaglomerular cells. What are these cells?

Ендотелій капілярів клубочка Endothelium of glomerular capillaries

Подоцити Podocytes

Юкстагломерулярні клітини Juxtaglomerular cells

Мезангіальні клітини Mesangial cells

Клітини щільної плями Dense spot cells

20 / 158
Хворий 67-ми років вживає з їжею переважно яйця, сало, вершкове масло, молоко, м’ясо. У крові виявлено холестерин 12,3 ммоль/л, загальні ліпіди - 8,2 г/л, підвищені фракції ліпопротеїнів низької щільності (ЛПНЩ). Яка гіпер-ліпопротеїнемія спостерігається у хворого? A 67-year-old patient eats mainly eggs, lard, butter, milk, meat. Cholesterol 12.3 mmol/l was found in the blood, total lipids - 8.2 g/l, increased fractions of low-density lipoproteins (LDL). What kind of hyperlipoproteinemia is observed in the patient?

Гіперліпопротеїнемія І типу Type I hyperlipoproteinemia

Гіперліпопротеїнемія IIb типу Hyperlipoproteinemia type IIb

Холестерин, гіперліпопротеїнемія Cholesterol, hyperlipoproteinemia

Гіперліпопротеїнемія ІІа типу Hyperlipoproteinemia type IIa

Гіперліпопротеїнемія IV типу Hyperlipoproteinemia type IV

21 / 158
У підлітка 12-ти років, який хворіє на бронхіальну астму, виник тяжкий напад астми: виражена експіраторна задишка, блідість шкірних покривів. Який вид порушення альвеолярної вентиляції має місце? A 12-year-old teenager suffering from bronchial asthma had a severe asthma attack: severe expiratory shortness of breath, pallor of the skin. What type of alveolar ventilation disorder occurs ?

Обструктивний Obstructive

Рестриктивний Restrictive

Торако-діафрагмальний Thoraco-diaphragmatic

Центральний Central

Нервово-м’язовий Neuromuscular

22 / 158
У хворого травма у ділянці передньої поверхні переднього драбинчастого м’яза. Функція якого нерва може бути порушена? The patient has an injury in the area of the anterior surface of the anterior scalene muscle. The function of which nerve may be impaired?

Плечове сплетення Brachial Plexus

Зворотній гортанний Reverse laryngeal

Додатковий Additional

Блукаючий Wandering

Діафрагмальний Aperture

23 / 158
Пацієнту, що хворіє на цукровий діабет та алергічний дерматит, лікар призначив фторований гормональний препарат в мазі. На питання хворого, про переваги призначеного препарату над маззю гідрокортизону лікар пояснив, що: To a patient suffering from diabetes and allergic dermatitis, the doctor prescribed a fluorinated hormonal drug in an ointment. To the patient's question about the advantages of the prescribed drug over hydrocortisone ointment, the doctor explained, what:

Коштує дешевше Costs cheaper

Діє короткочасно Effective for a short time

Призначений препарат практично немає резорбтивної дії The prescribed drug has practically no resorptive effect

Посилює синтез інсуліну Strengthens insulin synthesis

Діє слабше Works weaker

24 / 158
На розтині у потиличній частці головного мозку виявлена порожнина 2,5x1,5 см, заповнена прозорою рідиною, стінка її гладенька, буруватого кольору. Який процес розвинувся у головному мозку? At autopsy, a 2.5x1.5 cm cavity was found in the occipital lobe of the brain, filled with a transparent liquid, its wall was smooth, brownish in color. What process developed in the brain ?

Сіре розм’якшення мозку Gray softening of the brain

Абсцес мозку Brain abscess

Кіста на місці крововиливу Cyst at the site of hemorrhage

Вада розвитку мозку Brain malformation

Кіста на місці сірого розм’якшення Cyst at the place of gray softening

25 / 158
Під час дослідження коронарних артерій виявлені атеросклеротичні бляшки з кальцинозом, що закривають просвіт на 1/3. У м’язі дрібні множинні білуваті прошарки сполучної тканини. Який процес виявлено в міокарді? During the examination of the coronary arteries, atherosclerotic plaques with calcinosis were detected, covering 1/3 of the lumen. In the muscle, there are small multiple whitish layers of connective tissue. What process was detected in the myocardium?

Дифузний кардіосклероз Diffuse cardiosclerosis

Тигрове серце Tiger Heart

Міокардит Myocarditis

Післяінфарктний кардіосклероз Postinfarction cardiosclerosis

Інфаркт міокарда Myocardial infarction

26 / 158
У хворого запалення легенів ускладнилось ексудативним плевритом. В якому з перелічених анатомічних утворень переважним чином може накопичуватися рідина? The patient's pneumonia was complicated by exudative pleurisy. In which of the listed anatomical formations can fluid accumulate predominantly?

sinus costomediastinalis pleurae sinus costomediastinalis pleurae

sinus obliquus pericardii sinus obliquus pericardii

sinus costodiaphragmaticus pleurae sinus costodiaphragmaticus pleurae

sinus transversus pericardii sinus transversus pericardii

sinus phrenicomediastinalis pleurae sinus phrenicomediastinalis pleurae

27 / 158
До інфекційного відділення поступив хворий з ознаками жовтяниці внаслідок зараження вірусом гепатиту. Який з перерахованих показників є суворо специфічним, що відрізняє паренхіматозну жовтяницю від решти? A patient was admitted to the infectious disease department with signs of jaundice due to infection with the hepatitis virus. Which of the listed indicators is strictly specific, distinguishing parenchymal jaundice from the rest?

Білірубінурія Bilirubinuria

Підвищення рівня АлАТ, АсАТ Increasing the level of ABP, ABP

Гіпербілірубінемія Hyperbilirubinemia

Уробілінурія Urobilinuria

Холалемія Cholalemia

28 / 158
У хворого з дихальною недостатністю рН крові 7,35. Визначення рСО. показало наявність гіперкапнії. При дослідженні рН сечі відзначається підвищення її кислотності. Яка форма порушення кислотно-основного стану в даному випадку? A patient with respiratory insufficiency has a blood pH of 7.35. The determination of pCO. showed the presence of hypercapnia. When examining the pH of urine, an increase in its acidity is noted. What form of acid-base disturbance state in this case?

Алкалоз газовий, компенсований Gas alkalosis, compensated

Алкалоз газовий, декомпенсований Gas alkalosis, decompensated

Ацидоз газовий, компенсований Gas acidosis, compensated

Ацидоз метаболічний, компенсований Metabolic acidosis, compensated

Ацидоз метаболічний, декомпенсований Metabolic acidosis, decompensated

29 / 158
В хірургічне відділення доставлено чоловіка 35 років з гнійною раною на шиї попереду трахеї ( в ділянці передвісцерального простору). Куди може розповсюджуватись інфекція, якщо хворому терміново не зроблять операцію? A 35-year-old man was brought to the surgical department with a purulent wound on the neck in front of the trachea (in the area of the previsceral space). Where can the infection spread if the patient is not operated urgently?

В грудну порожнину - переднє середостіння Into the chest cavity - anterior mediastinum

В ретровісцеральний простір Into the retrovisceral space

В грудну порожнину - в заднє середостіння In the chest cavity - in the posterior mediastinum

В грудну порожнину - в середнє середостіння In the chest cavity - in the middle mediastinum

В надгрудинний міжапоневротичний простір In the suprasternal interaponeurotic space

30 / 158
Який стан може розвинутися через 15-30 хвилин після повторного введення антигену внаслідок підвищеного рівня антитіл, переважно ІдЕ, які адсорбуються на поверхні клітин-мішеней - тканинних базофілів (тучних клітин) та базофілів крові? What condition can develop 15-30 minutes after re-introduction of an antigen due to an increased level of antibodies, mainly IdE, which are adsorbed on the surface of target cells - tissue basophils (fat cells) and blood basophils?

Імунно-комплексна гіперчутливість Immune complex hypersensitivity

Анафілаксія Anaphylaxis

Гіперчутливість уповільненого типу Delayed hypersensitivity

Антитіло-залежна цитотоксичність Antibody-dependent cytotoxicity

Сироваткова хвороба Serum sickness

31 / 158
До медико-генетичної консультації звернулися батьки хворої дівчинки 5 років. Після дослідження каріотипу виявили 46 хромосом. Одна з хромосом 15-ї пари була довша від звичайної, тому що до неї приєдналася хромосома з 21-ї пари. Який вид мутації має місце в цієї дівчинки? Parents of a sick 5-year-old girl turned to medical and genetic counseling. After a karyotype study, 46 chromosomes were found. One of the chromosomes of the 15th pair was longer than usual, because a chromosome from pair 21 joined her. What kind of mutation does this girl have?

Інверсія Inversion

Делеція Deletion

Нестача Shortage

Транслокація Translocation

Дуплікація Duplication

32 / 158
У хлопчика 2-х років діагностована хвороба Дауна. Які зміни в хромосомах можуть бути причиною цієї хвороби? A 2-year-old boy is diagnosed with Down's disease. What changes in chromosomes can be the cause of this disease?

Моносомія по Х-хромосомі Monosomy on the X-chromosome

Трисомія по 18-й хромосомі Trisomy on the 18th chromosome

Трисомія по 13-й хромосомі Trisomy on the 13th chromosome

Трисомія по 21-й хромосомі Trisomy on the 21st chromosome

Трисомія по Х-хромосомі Trisomy on the X-chromosome

33 / 158
93 Досліджують процеси тепловіддачі у роздягненої людини при кімнатній температурі. З'ясовано, що за таких умов найбільша кількість тепла віддається шляхом: 93 The processes of heat transfer in a naked person at room temperature are studied. It was found that under such conditions, the largest amount of heat is released by:

Конвекції Convections

Випаровування Evaporation

- -

Теплорадіацїї Thermal radiation

Теплопроведення Heat conduction

34 / 158
У зв’язку з вираженим больовим синдромом хворому призначено наркотичний анальгетик. Вкажіть обраний препарат: Due to severe pain, the patient is prescribed a narcotic analgesic. Specify the selected drug:

Анальгін Analgin

Індометацин Indomethacin

Морфін Morphine

Німесулід Nimesulide

Димексид Dimexide

35 / 158
В матеріалі, взятому від хворої людини, знайдено декілька видів мікроорганізмів (стафілококи та стрептококи різних видів), які стали причиною захворювання. Як називається такий вид інфекції? In the material taken from a sick person, several types of microorganisms (staphylococci and streptococci of various species) were found that caused the disease. What is this type of infection called?

Суперінфекція Superinfection

Коінфекція Coinfection

Змішана інфекція Mixed infection

Реінфекція Reinfection

Вторинна інфекція Secondary infection

36 / 158
Хворий 30-ти років після перенесеного вірусного гепатиту В став скаржитися на тривалі носові кровотечі. Призначення якого засобу буде найбільш раціональним для корекції цього стану? A 30-year-old patient began to complain of prolonged nosebleeds after suffering from viral hepatitis B. What remedy would be the most rational to correct this condition?

Дипіридамол Dipyridamole

Фолієва кислота Folic acid

Аспаркам Asparkam

Вікасол Vikasol

Фраксіпарин Fraxiparin

37 / 158
Активація ряду факторів системи гемостазу здійснюється шляхом приєднання до них іонів кальцію. Наявність якого структурного компонента у їх складі забезпечує приєднання іонів кальцію? Activation of a number of factors of the hemostasis system is carried out by attaching calcium ions to them. The presence of which structural component in their composition ensures the attachment of calcium ions?

Гідроксипролін Hydroxyproline

у-оксимасляна кислота y-oxybutyric acid

у-карбоксиглутамінова кислота y-carboxyglutamic acid

у-аміномасляна кислота y-aminobutyric acid

Моноамінодикарбонові кислоти Monoaminodicarboxylic acids

38 / 158
Екзофтальм, що спостерігається при тиреотоксикозі, обумовлений накопиченням в ретробульбарній тканині речовин, які мають високу здатністю зв’язувати воду. Які це речовини? Exophthalmos observed in thyrotoxicosis is caused by the accumulation of substances in the retrobulbar tissue that have a high ability to bind water. What are these substances?

Холестерин Cholesterol

Фосфоліпіди Phospholipids

Креатин Creatine

АТФ ATP

Глікозаміноглікани Glycosaminoglycans

39 / 158
У дівчинки 3-х років з затримкою психічного розвитку діагностовано сфінголіпідоз (хвороба Німана-Піка). Порушення синтезу якої речовини спостерігається при цьому? A 3-year-old girl with mental retardation was diagnosed with sphingolipidosis (Niemann-Pick disease). Violation of the synthesis of which substance is observed?

Гангліозиди Gangliosides

Цераміди Ceramides

Сфінгомієліназа Sphingomyelinase

Глікозилтрансфераза Glycosyltransferase

Сфінгозин Sphingosine

40 / 158
У людини через 10 хвилин після початку інтенсивної фізичної роботи кількість еритроцитів у крові збільшилася з 4,0*1012/л до 4,5*1012/л. Що є основною причою цього? In a person, 10 minutes after the start of intensive physical work, the number of erythrocytes in the blood increased from 4.0*1012/l to 4.5*1012/l. What is the main reason for this?

Пригнічення руйнування еритроцитів Inhibition of erythrocyte destruction

Втрата води організмом Body water loss

Вихід еритроцитів з депо Output of erythrocytes from the depot

Збільшення хвилинного об’єму крові Increase in minute blood volume

Активація еритропоезу Activation of erythropoiesis

41 / 158
У жінки, яка відпочивала на дачі, відразу після укусу оси виник біль, через кілька хвилин на шкірі в місці укусу з’явився пухир, еритема і сильне свербіння, а ще через деякий час -кропив’янка, експіраторна задишка. Внаслідок дії яких факторів у хворої розвинулась експіраторна задишка? A woman who was vacationing in the country felt pain immediately after being bitten by a wasp, after a few minutes a blister appeared on the skin at the site of the bite, erythema and severe itching, and after some time - urticaria, expiratory shortness of breath. As a result of what factors did the patient develop expiratory shortness of breath?

Фактор Хагемана Hageman Factor

Гістамін Histamine

Адреналін Adrenaline

Норадреналін Noradrenaline

Лізосомальні ферменти Lysosomal enzymes

42 / 158
У хворого, який скаржиться на поліурію і полідипсію, знайдено цукор в сечі. Вміст цукру в плазмі крові у нормі. З чим пов'язаний механізм глюкозурії у хворого? Sugar was found in the urine of a patient who complains of polyuria and polydipsia. The content of sugar in the blood plasma is normal. What is the mechanism of glucosuria in the patient?

Порушення фільтрації глюкози в клубом новому відділі нефрону Disturbance of glucose filtration in the club new division of the nephron

Пперпродукція глюкокортикоїдів наднирниками Overproduction of glucocorticoids by adrenal glands

Порушення реабсорбції глюкози в канальцях нефрону Disturbance of glucose reabsorption in nephron tubules

Інсулінорезистентність рецепторів клітин Insulin resistance of cell receptors

Недостатня продукція інсуліну підшлунковою залозою Insufficient production of insulin by the pancreas

43 / 158
На електронній фотографії представлена органела, що являє собою великий поліпротеазний комплекс, що складається з трубкоподібної та двох регуляторних частин, які розташовані на обох кінцях органели. Остання виконує функцію протеолізу. Назвіть цю органелу. The electronic photo shows an organelle, which is a large polyprotease complex consisting of a tubular and two regulatory parts, which are located at both ends of the organelle. The latter performs the function of proteolysis . Name this organelle.

Включення Inclusion

Протеасома. Proteasome.

Комплекс Гольджі Golgi Complex

Рибосома Ribosome

Центріоль Centriol

44 / 158
У чоловіка виявлене захворювання, яке зумовлене домінантним геном, локалізованим у Х-хромосомі. У кого із дітей буде це захворювання, якщо дружина здорова? The husband has been diagnosed with a disease caused by a dominant gene located on the X-chromosome. Which of the children will have this disease if the wife is healthy?

У половини синів Half sons

У всіх дітей All children have

Тільки у дочок Only in daughters

У половини дочок Half daughters

Тільки у синів Only sons

45 / 158
У пацієнта, що прибув з ендемічного за малярією району, підвищилася температура тіла, відзначається головний біль, озноб, загальне нездужання -симптоми, що характерні й для звичайної застуди. Які лабораторні дослідження необхідно провести, щоб підтвердити або спростувати діагноз 'малярія'? The patient, who arrived from a malaria-endemic area, has an elevated body temperature, headache, chills, and general malaise - symptoms that are also characteristic of a common cold. What laboratory tests should be conducted to confirm or deny the diagnosis of 'malaria'?

Мікроскопія мазків крові Microscopy of blood smears

Дослідження спинномозкової рідини Research of cerebrospinal fluid

Аналіз сечі Urine analysis

Мікроскопія пунктату червоного кісткового мозку Red bone marrow punctate microscopy

Дослідження пунктату лімфовузлів Study of punctate lymph nodes

46 / 158
При повторному введенні алергену починається виділення гістаміну тучними клітинами крові. До якого рівня реактивності відноситься така відповідь організму? When the allergen is re-introduced, the release of histamine by blood mast cells begins. To what level of reactivity does this response of the body belong?

Субклітинний Subcellular

Органний Organic

Системний System

Молекулярний Molecular

Клітинний Cellular

47 / 158
У хлопчика 7 р., із дрібнокрапчастим яскраво-рожевим висипом на гіперемованому фоні шкіри лоба, шиї, внизу живота, підколінних ямках, носо-губний трикутник блідий. В ротоглотці — відмежована яскраво-червона гіперемія, мигдалики набряклі, рихлі, в лакунах є гній, малиновий язик. Шийні лімфовузли збільшені, щільні, болючі. Поставити діагноз: A 7-year-old boy with a small-dotted, bright pink rash on a hyperemic background of the skin of the forehead, neck, lower abdomen, popliteal fossa, the nasolabial triangle is pale. oropharynx - demarcated bright red hyperemia, tonsils are swollen, loose, there is pus in lacunae, crimson tongue. Cervical lymph nodes are enlarged, dense, painful. Make a diagnosis:

Дифтерія Diphtheria

Скарлатина Scarlatina

Інфекційний мононуклеоз Infectious mononucleosis

Краснуха Krasnukha

Коклюш Whooping cough

48 / 158
У жінки 45-ти років артеріальна гіпертензія с високою концентрацією ангіотензину-2 у крові. Який з перерахованих антигіпертензивних засобів найбільш показаний у цьому випадку? A 45-year-old woman has arterial hypertension with a high concentration of angiotensin-2 in the blood. Which of the listed antihypertensive drugs is most indicated in this case?

Метопролол Metoprolol

Празозин Prazozin

Верапаміл Verapamil

Лізиноприл Lisinopril

Резерпін Reserpin

49 / 158
Відомо, що робота в шахті пов’язана з вдиханням значної кількості вугільного пилу. В яких клітинах легень можна виявити вугільний пил? It is known that working in a mine involves inhaling a significant amount of coal dust. In which lung cells can coal dust be detected?

Ендотеліоцитах капілярів Capillary endotheliocytes

Респіраторних епітеліоцитах Respiratory epitheliocytes

Секреторних епітеліоцитах Secretory epitheliocytes

Перицитах капілярів Capillary pericytes

Альвеолярних макрофагах Alveolar macrophages

50 / 158
Хворий 40-ка років страждає на бронхіальну астму та скаржиться на тахікардію протягом тривалого часу. Вкажіть найбільш оптимальний в даній ситуації препарат для купірування бронхоспазму: A 40-year-old patient suffers from bronchial asthma and complains of tachycardia for a long time. Specify the most optimal drug for stopping bronchospasm in this situation:

Адреналіну гідрохлорид Adrenaline hydrochloride

Сальбутамол Salbutamol

Ефедрину гідрохлорид Ephedrine hydrochloride

Ізадрин Izadrin

Орципреналіну сульфат Orciprenaline sulfate

51 / 158
Хворий доставлений в хірургічне відділення з різаною раною передньої поверхні плеча у нижній третині. В нього порушена функція згинання в плечовому і ліктьовому суглобах. Це є результатом ушкодження такого м’язу: The patient was brought to the surgical department with a cut wound of the front surface of the shoulder in the lower third. He has impaired flexion function in the shoulder and elbow joints. This is the result of damage to such a muscle yazu:

Дельтоподібний м’яз Deltoid

Дзьобоподібно-плечевий м’яз Proboscis-brachial muscle

Трьохголовий м’яз плеча Triceps

Двоголовий м’яз плеча Biceps brachii

Ліктьовий м’яз Elbow muscle

52 / 158
У хворого на цукровий діабет розвинулась діабетична нефропатія з розвитком уремії. Швидкість клубочкової фільтрації 9 мл/хв. Який найбільш імовірний механізм зниження швидкості клубочкової фільтрації і розвитку хронічної ниркової недостатності у пацієнта? A patient with diabetes developed diabetic nephropathy with the development of uremia. The glomerular filtration rate is 9 ml/min. What is the most likely mechanism for the reduction of the glomerular filtration rate and the development of chronic renal failure in the patient?

Зниження системного артеріального тиску Decreasing systemic blood pressure

Зменшення маси діючих нефронів Decreasing mass of functioning nephrons

Розвиток ацидозу в тканинах Development of acidosis in tissues

Спазм артеріол Arteriolar spasm

Закупорка просвіту канальців нефрону гіаліновими циліндрами Clogging of lumen of nephron tubules with hyaline cylinders

53 / 158
У хворої на бронхіальну астму вірусне інфікування спровокувало астматичний статус зі смертельним наслідком. При гістологічному дослідженні легень виявлено: спазм і набрякання бронхіол, в їх стінках виражена інфільтрація лімфоцитами, еозинофілами та іншими лейкоцитами, а також дегрануляція лаброцитів. Який механізм гіперчутливості лежить в основі зазначених змін? In a patient with bronchial asthma, a viral infection provoked an asthmatic status with a fatal outcome. Histological examination of the lungs revealed: spasm and swelling of the bronchioles, marked infiltration of lymphocytes and eosinophils in their walls and other leukocytes, as well as degranulation of labrocytes. What mechanism of hypersensitivity underlies these changes?

Імунозумовлений клітинний цитоліз Immune-induced cellular cytolysis

Реагінова реакція Reagin reaction

Запальний Incendiary

Імунокомплексний Immunocomplex

Аутоімунний Autoimmune

54 / 158
На розтиш тїла жінки з середньої Азії, що тривалий час страждає на виснажуючу лихоманку, були виявлені збільшені' печінка і селезінка аспідно-чорного кольору. Кістковий мозок гіперплазований, такого ж кольору. Кора головного мозку сіро-димчатого кольору. Для якого захворювання характерні такі зміни? An autopsy of a woman from Central Asia, suffering from a debilitating fever for a long time, revealed an enlarged liver and an aspid-black spleen. The bone marrow is hyperplastic, such the same color. The cortex of the brain is gray-smoky in color. What disease is characterized by such changes?

Малярія Malaria

Сепсис Sepsis

Гепатит Hepatitis

Висипний тиф Typhoid

СНЩ TNS

55 / 158
14. У лабораторному експерименті на собаці вивчали будову центральних від ділів слухової сенсорної системи. Була зруйнована одна з структур середнього мозку. Собака втратив орієнтувальний рефлекс на звукові сигнали. Яка структура була зруйнована? 14. In a laboratory experiment on a dog, the structure of the central parts of the auditory sensory system was studied. One of the structures of the midbrain was destroyed. The dog lost its orientation reflex to sound signals. What was the structure destroyed?

Чорна речовина Black Matter

Верхні горбики чотиригорбикового тіла Upper tubercles of quadrituberous body

Ядра ретикулярної формації Nuclei of reticular formation

Нижні горбики чотиригорбикового тіла Inferior tubercles of quadrituberous body

Червоне ядро Red Core

56 / 158
Чоловіку 40-ка років за результатами діагностичних тестів зробили лімфографію органів грудної порожнини. Хірург встановив, що пухлина вразила орган, з лімфатичних судин якого лімфа безпосередньо переходить в грудну протоку. Який це орган? Based on the results of diagnostic tests, a 40-year-old man underwent lymphography of the organs of the chest cavity. The surgeon established that the tumor affected the organ from whose lymphatic vessels the lymph directly passes into the thoracic duct . What organ is this?

Стравохід Esophagus

Осердя Hearts

Трахея Trachea

Серце Heart

Лівий головний бронх Left main bronchus

57 / 158
В лікарню надійшов потерпілий з рваною раною гомілки, яка виникла внаслідок того, що його покусала хвора на сказ тварина. Яку вакцину необхідно ввести для попередження сказу? The victim came to the hospital with a torn leg wound, which occurred as a result of being bitten by an animal with rabies. What vaccine should be administered to prevent rabies?

Антирабічна вакцина Anti-rabies vaccine

TABte TABte

АДП ADP

БЦЖ BCG

АКДП AKDP

58 / 158
У людини внаслідок патологічного процесу збільшена товщина альвеолокапіляр-ної мембрани. Безпосереднім наслідком цього буде зменшення у людини: Due to a pathological process, the thickness of the alveolocapillary membrane has increased in a person. The direct consequence of this will be a decrease in a person:

Дифузійної здатності легень Diffusing capacity of lungs

Резервного об’єму видиху Exhalation reserve volume

Альвеолярної вентиляції легень Alveolar lung ventilation

Хвилинного об’єму дихання Minute respiratory volume

Кисневої ємності крові Blood oxygen capacity

59 / 158
В експерименті подразнюють гілочки симпатичного нерва, які інервують серце. Це призвело до збільшення сили серцевих скорочень, тому що через мембрану типових кардіоміоцитів збільшився: In the experiment, branches of the sympathetic nerve, which innervate the heart, are irritated. This led to an increase in the strength of heart contractions, because through the membrane of typical cardiomyocytes, increased:

Вихід іонів калію Output of potassium ions

Вхід іонів кальцію та калію Input of calcium and potassium ions

Вхід іонів кальцію Input of calcium ions

Вихід іонів кальцію Output of calcium ions

Вхід іонів калію Input of potassium ions

60 / 158
Застосування антикоагулянтного засобу викликало крововиливи на слизових оболонках, макрогематурію, зменшення часу згортання крові. Введення протаміну сульфату усунуло ці порушення. Який засіб було призначено? The use of an anticoagulant caused hemorrhages on the mucous membranes, macrohematuria, a decrease in blood clotting time. The introduction of protamine sulfate eliminated these disorders. What remedy was prescribed?

Фібринолізин Fibrinolysin

Натрію цитрат Sodium Citrate

Гепарин Heparin

Неодикумарин Neodicoumarin

Вікасол Vikasol

61 / 158
У хворого 47-ми років виникла кишкова коліка на фоні гіпертонічної хвороби. Засоби якої з перерахованих груп найбільш доцільно використати для її купірування у даній ситуації? A 47-year-old patient developed intestinal colic on the background of hypertension. Which of the listed groups of drugs is the most appropriate to use to stop it in this situation?

Симпатоміметики Sympathomimetics

Міотропні спазмолітики Myotropic antispasmodics

Адреноміметики Adrenomimetics

Антихолінестеразні засоби Anticholinesterase drugs

М-холиноміметики M-cholinomimetics

62 / 158
Чоловік 65-ти років раптово втратив зір на одне око в зв'язку з відшаруванням сітківки. Хворому була проведена енуклеація. Під час гістологічного дослідження видаленого очі в сітківці і судинній оболонці виявлені комплекси атипових клітин з вираженим поліморфізмом клітин і ядер, з помірною кількістю мітозів, враховуючи патологічні. У цитоплазмі клітин і міжклітинної середовищі виявляється пігмент коричневого кольору, що дає позитивну реакцію ДОФА. Реакція Перлса негативна. Який найбільш ймовірний діагноз? A 65-year-old man suddenly lost vision in one eye due to retinal detachment. The patient underwent enucleation. During histological examination of the removed eye, the retina and complex of atypical cells with pronounced polymorphism of cells and nuclei, with a moderate number of mitoses, including pathological ones, were found in the choroid. In the cytoplasm of the cells and the intercellular medium, a brown pigment is detected, which gives a positive DOPA reaction. The Perls reaction is negative. What is the most likely diagnosis?

Цістоцеркоз Cystocerciasis

пігментний невус pigmented nevus

Хвороба Вільсона-Коновалова Wilson-Konovalov disease

крововилив hemorrhage

меланома melanoma

63 / 158
У вагітної жінки виявлені IgM до вірусу краснухи, на підставі чого акушер-гінеколог рекомендував перервати вагітність через високу ймовірність тератогенного впливу на плід. Важливим було виявлення саме Ig M, так як імуноглобуліни цього класу: IgM to the rubella virus was detected in a pregnant woman, on the basis of which the obstetrician-gynecologist recommended terminating the pregnancy due to the high probability of a teratogenic effect on the fetus. It was important to detect Ig M, as immunoglobulins of this class:

Мають найбільшу молекулярну вагу Have the highest molecular weight

Можуть долати плацентарний бар'єр Can cross the placental barrier

Є основним фактором противірусного захисту Is the main factor of antivirus protection

Є показником свіжого зараження Is an indicator of recent infection

Пов'язані з анафілактичними реакціями Associated with anaphylactic reactions

64 / 158
У хворого, внаслідок тривалого хронічного захворювання головного мозку, виникли мимовільні рухи, порушився тонус м’язів тулуба. На порушення якого провідного шляху вказують ці симптоми? As a result of a long-term chronic disease of the brain, the patient developed involuntary movements, the tone of the muscles of the trunk was disturbed. What conduction path are these symptoms indicative of?

Tractus spinothalamicus Tractus spinothalamicus

Tractus tectospinalis Tractus tectospinalis

Tractus corticospinalis Tractus corticospinalis

Tractus corticonuclearis Tractus corticonuclearis

Tractus rubrospinalis Tractus rubrospinalis

65 / 158
У хворого, який знаходиться у клініці з ознаками отруєння ртуттю, у нирках відмічаються наступні процеси: вогнищеві некротичні зміни канальців головних відділів, набряк, лейкоцитарна інфільтрація та геморагії інтерстицію, венозний застій. Який стан розвився у хворого? In a patient who is in the clinic with signs of mercury poisoning, the following processes are noted in the kidneys: focal necrotic changes in the tubules of the main sections, edema, leukocyte infiltration and interstitial hemorrhages, venous stasis. What condition has the patient developed?

Гострий гломерулонефрит Acute glomerulonephritis

Гострий пієлонефрит Acute pyelonephritis

Хронічний пієлонефрит Chronic pyelonephritis

Гострий некротичний нефроз Acute necrotic nephrosis

Хронічна ниркова недостатність Chronic renal failure

66 / 158
У хворого висока температура, задуха, біль у правій частині грудної клітини. Плевральна пункція дала 700 мл в'язкої рідини жовто-зеленого кольору. Який патологічний процес розвився у плевральній порожнині? The patient has a high temperature, shortness of breath, pain in the right side of the chest. Pleural puncture yielded 700 ml of yellow-green viscous liquid. What pathological process developed in pleural cavity?

Емпієма плеври Empyema of the pleura

Серозний плеврит Serous pleurisy

Геморрагичний плеврит Hemorrhagic pleurisy

Бронхопневмонія Bronchopneumonia

Карциноматоз плеври Carcinomatosis of the pleura

67 / 158
Онкологічному хворому призначили препарат метотрексат, до якого з часом клітини-мішені пухлини втратили чутливість. Експресія гену якого ферменту при цьому змінюється? A cancer patient was prescribed the drug methotrexate, to which over time the target cells of the tumor lost sensitivity. The gene expression of which enzyme changes?

Тиміназа Thyminase

Дезаміназа Deaminase

Фолатдекарбоксилаза Folate decarboxylase

Фолатоксидаза Folate oxidase

Дегідрофолатредуктаза Dehydrofolate reductase

68 / 158
Невпізнання хворим предметів при їх обмацуванні виникло після черепно-мозкової травми. Який відділ мозку ушкоджено? The patient's failure to recognize objects when feeling them occurred after a brain injury. What part of the brain is damaged?

Прецентральна звивина Precentral gyrus

Постцентральна звивина Postcentral gyrus

Скронева частка Temporal lobe

Потилична частка Occipital lobe

Мозочок Cerebellum

69 / 158
При патологічних процесах, які супроводжуються гіпоксією, відбувається неповне відновлення молекули кисню в дихальному ланцюзі і накопичення пероксиду водню. Вкажіть фермент, який забезпечує його руйнування: In pathological processes that are accompanied by hypoxia, there is an incomplete restoration of the oxygen molecule in the respiratory chain and the accumulation of hydrogen peroxide. Specify the enzyme that ensures its destruction:

Каталаза Catalase

Сукцинатдегідрогеназа Succinate dehydrogenase

Аконітаза Aconitase

Цитохромоксидаза Cytochrome oxidase

Кетоглутаратдегідрогеназа Ketoglutarate dehydrogenase

70 / 158
В анамнезі жінки зазначено три викидні, внаслідок четвертої вагітності народилася дитина з ураженням центральної нервової системи та очей, збільшенням лімфовузлів та селезінки. Відомо, що дома у жінки живуть дві кішки. Мікроскопічне дослідження мазків крові та пунктатів лімфовузлів виявило в клітинах тільця у формі півмісяця, у яких один кінець загострений і має утворення у вигляді присоски, а інший - заокруглений. Який паразит виявлений у жінки? The woman's history shows three miscarriages, as a result of the fourth pregnancy, a child was born with damage to the central nervous system and eyes, an increase in the lymph nodes and spleen. It is known that the woman lives at home with two cats. Microscopic examination of blood smears and punctates of lymph nodes revealed crescent-shaped bodies in the cells, in which one end is pointed and has a suction-shaped formation, and the other is rounded. What parasite was found in a woman?

Balantidium coli Balantidium coli

Toxoplasma gondii Toxoplasma gondii

Lamblia intestinalis Lamblia intestinalis

Trichomonas hominis Trichomonas hominis

Plasmodium vivax Plasmodium vivax

71 / 158
Жінка 45-ти років, перукар, скаржиться на болі у ногах, що з’являються після роботи, ввечері та вночі. При огляді хворої виявлено варикозне розширені вени на присередній поверхні гомілки та стегна. Яка вена та її безпосередні протоки розширені? A 45-year-old woman, a hairdresser, complains of pain in her legs that appear after work, in the evening and at night. During the patient's examination, varicose veins were found on on the medial surface of the lower leg and thigh. Which vein and its immediate ducts are dilated?

Велика підшкірна Large subcutaneous

Стегнова Thigh

Мала підшкірна Small subcutaneous

Передня великогомілкова вена Anterior tibial vein

Глибока вена стегна Deep thigh vein

72 / 158
При обстежені пацієнта встановили сильний, врівноважений, інертний тип вищої нервової діяльності за Павловим. Якому темпераменту за Гіппократом відповідає пацієнт? When examining the patient, a strong, balanced, inert type of higher nervous activity according to Pavlov was established. What temperament does the patient correspond to according to Hippocrates?

Холерик Choleric

- -

Флегматик Phlegmatic

Сангвінік Sanguine

Меланхолік Melancholic

73 / 158
У чоловіка 30-ти років перед операцією визначили групу крові. Кров резус-позитивна. Реакція аглютинації еритроцитів не відбулася зі стандартними сироватками груп 0 (І), А (II), В (III). Досліджувана кров належить до групи: The blood group of a 30-year-old man was determined before the operation. The blood is Rh-positive. The erythrocyte agglutination reaction did not occur with standard sera of groups 0 (I), A ( II), B (III). The studied blood belongs to the group:

- -

АВ (IV) AB (IV)

0 (І) 0 (And)

В (III) In (III)

А (II) A (II)

74 / 158
У підлітка 13-ти років під час проведення рентгенографічного дослідження кульшового суглоба виявлено зону просвітлення шириною 3 мм між голівкою та діафізом стегнової кістки. Як слід оцінити таку ситуацію? A 13-year-old teenager was found to have a 3 mm-wide luminal zone between the head and diaphysis of the femur during X-ray examination of the hip joint. How should such a situation be assessed?

Як норму (незавершений процес окостеніння) As normal (incomplete ossification process)

Як вивих головки стегнової кістки As a dislocation of the femoral head

Як артефакт на рентгенологічній плівці Like an artifact on x-ray film

Як тріщину шийки стегнової кістки Like a femoral neck crack

Як перелом шийки стегнової кістки As a femoral neck fracture

75 / 158
У жінки з III (Б), Rh- групою крові народилась дитина з II (A) групою крові. У дитини діагностовано гемолітичну хворобу новонародженого внаслідок резус-конфлікту. Яка група крові за системою АБо та резус-належність можливі у батька? A woman with III (B), Rh blood group gave birth to a child with II (A) blood group. The child was diagnosed with hemolytic disease of the newborn as a result of Rhesus conflict. What blood group according to the ABO system and Rh-affinity are possible for the father?

II (A), Rh- II (A), Rh-

II (A), Rh+ II (A), Rh+

I (O), Rh+ I (O), Rh+

I (O), Rh- I (O), Rh-

III (Б), Rh+ III (B), Rh+

76 / 158
Одним із факторів, що призводить до ожиріння, є пригнічення швидкості окислення жирних кислот внаслідок: One of the factors leading to obesity is inhibition of the rate of fatty acid oxidation due to:

Зниження вмісту карнітину Carnitine reduction

Через недостатність синтезу фосфо-ліпідів Due to insufficient synthesis of phospho-lipids

Через недостатність вуглеводів у харчуванні Due to insufficient carbohydrates in the diet

Через недостатність холіну Due to choline deficiency

Надмірного вживання жирної їжі Excessive consumption of fatty food

77 / 158
Хворий після перенесеного епідемічного паротиту схуднув, постійно відчуває спрагу, п'є багато води, відмічає часте сечовиділення, підвищений апетит, шкірний свербіж, слабкість, фурункульоз. У крові: глюкоза - 16 ммоль/л, кетонових тіл - 100 мкмоль/л; глюкозурія. Яке захворювання розвинулось у пацієнта? After suffering from epidemic mumps, the patient has lost weight, is constantly thirsty, drinks a lot of water, notes frequent urination, increased appetite, skin itching, weakness, furunculosis. In the blood : glucose - 16 mmol/l, ketone bodies - 100 μmol/l; glucosuria. What disease did the patient develop?

Інсулінонезалежний цукровий діабет Insulin-dependent diabetes

Нецукровий діабет Diabetes insipidus

Цукровий діабет недостатнього харчування Diabetes undernutrition

Стероїдний діабет Steroid diabetes

Інсулінозалежний цукровий діабет Insulin-dependent diabetes

78 / 158
Під час вивчення фаз мітотичного циклу корінця цибулі знайдено клітину, в якій хромосоми лежать в екваторіальній площині, утворюючи зірку. На якій стадії мітозу перебуває клітина? While studying the phases of the mitotic cycle of an onion root, a cell was found in which the chromosomes lie in the equatorial plane, forming a star. At what stage of mitosis is the cell?

Профаза Prophase

Інтерфаза Interphase

Анафаза Anaphase

Телофаза Telophase

Метафаза Metaphase

79 / 158
Після попередньої сенсибілізації експериментальній тварині підшкірно ввели дозу антигену. У місці ін'єкції розвинулось фібринозне запалення з альтерацією стінок судин основної речовини та волокнистих структур сполучної тканини у вигляді мукоїдного та фібриноїдного набухання і некрозу. Яка імунологічна реакція має місце? After preliminary sensitization, the experimental animal was subcutaneously injected with a dose of antigen. At the injection site, fibrinous inflammation developed with alteration of the vessel walls of the main substance and fibrous structures of connective tissue in the form of mucoid and fibrinoid swelling and necrosis. What immunological reaction takes place?

Гіперчутливість сповільненого типу Delayed type hypersensitivity

Гранулематоз Granulomatosis

Гіперчутливість негайного типу Immediate Hypersensitivity

Реакція трансплантаційного імунітету Reaction of transplant immunity

Нормергічна реакція Normergic reaction

80 / 158
У гістологічному препараті паренхіма органа представлена лімфоїдною тканиною, яка утворює лімфатичні вузлики; останні розташовуються дифузно і містять центральну артерію. Яке анатомічне утворення має таку морфологічну будову? In the histological preparation, the parenchyma of the organ is represented by lymphoid tissue that forms lymph nodes; the latter are located diffusely and contain a central artery. What anatomical formation has such a morphological structure?

Мигдалик Amygdalik

Червоний кістковий мозок Red bone marrow

Лімфатичний вузол Lymph node

Селезінка Spleen

Тимус Thymus

81 / 158
У пацієнта виявлено різке розширення підшкірних вен в ділянці передньої черевної стінки навколо пупка. Підвищений тиск у якій судині сприяє цій симптоматиці? The patient has a sharp expansion of the subcutaneous veins in the area of the anterior abdominal wall around the navel. Increased pressure in which vessel contributes to these symptoms?

V mesenterica superior V mesenterica superior

V portae hepatis V portae hepatis

V cava inferior V cava inferior

V mesenterica inferior V mesenterica inferior

V cava superior V cava superior

82 / 158
У хворого виявлено зниження вмісту іонів магнію, які потрібні для прикріплення рибосом до гранулярної ендоплазматичної сітки. Відомо, що це призводить до порушення біосинтезу білка. Який саме етап біосинтезу білка буде порушено? The patient was found to have a decrease in the content of magnesium ions, which are required for the attachment of ribosomes to the granular endoplasmic reticulum. It is known that this leads to a violation of protein biosynthesis. What exactly is the stage of protein biosynthesis will be violated?

Реплікація Replication

Трансляція Broadcast

Термінація Termination

Транскрипція Transcription

Активація амінокислот Activation of amino acids

83 / 158
До токсикологічного відділення госпіталізовано жінку з отруєнням невідомою речовиною. Яка група лікарських засобів може бути застосована для зменшення всмоктування і надходження отрути в організм? A woman was hospitalized with poisoning by an unknown substance. What group of drugs can be used to reduce the absorption and entry of poison into the body?

Нейролептики Neuroleptics

Адсорбенти Adsorbents

Антиоксиданти Antioxidants

Антихолінестеразні Anticholinesterases

Органічні нітрати Organic nitrates

84 / 158
У хворого пухлиною пошкоджено піраміди довгастого мозку. У якому з провідних шляхів порушиться проведення нервових імпульсів? The patient's tumor has damaged the pyramids of the medulla oblongata. In which of the leading pathways will the conduction of nerve impulses be disrupted?

Tr. corticospinalis Tr. corticospinalis

Tr. spinocerebellaris Tr. spinocerebellaris

Tr. corticopontinus Tr. corticopontinus

Tr. dentatorubralis Tr. dentatorubralis

Tr. corticonuclearis Tr. corticonuclearis

85 / 158
Під час оформлення дитини до школи для вирішення питання про необхідність ревакцинації поставлена проба Манту, яка виявилася негативною. Про що свідчить даний результат проби? During the registration of the child to school, to resolve the issue of the need for revaccination, a Mantoux test was performed, which turned out to be negative. What does this test result indicate?

Про наявність антитіл до туберкульозних бактерій About the presence of antibodies to tuberculosis bacteria

Про відсутність антитоксичного імунітету до туберкульозу About lack of antitoxic immunity to tuberculosis

Про відсутність клітинного імунітету до туберкульозу About lack of cellular immunity to tuberculosis

Про наявність клітинного імунітету до туберкульозу On the presence of cellular immunity to tuberculosis

Про відсутність антитіл до туберкульозних бактерій On the absence of antibodies to tuberculosis bacteria

86 / 158
У хірургічне відділення лікарні був прийнятий хворий з підозрою на абсцес печінки. Хворий тривалий час знаходився у відрядженні в одній з африканських країн і неоднарозово хворів на гостре шлунково-кишкове захворювання. Яке протозойне захворювання може бути в хворого? A patient was admitted to the surgical department of the hospital with suspicion of a liver abscess. The patient was on a business trip in one of the African countries for a long time and repeatedly suffered from an acute gastrointestinal disease What kind of protozoan disease can the patient have?

Малярія Malaria

Амебіаз Amebiasis

Токсоплазмоз Toxoplasmosis

Трипаносомоз Trypanosomosis

Лейшманіоз Leishmaniasis

87 / 158
У пацієнта порушена функція нирок. Для перевірки стану фільтраційної здатності нирок йому призначено визначення кліренсу: The patient has impaired kidney function. In order to check the state of the filtration capacity of the kidneys, he is assigned clearance determination:

Глутаміну Glutamine

Сечової кислоти Uric acid

Гідрокарбонату Hydrocarbonate

!ндолу !ndolu

Креатиніну Creatinine

88 / 158
Обстеження пацієнта з високим артеріальним тиском виявило у нього вторинну артеріальну гіпертензію. З’ясовано, що причиною такого стану пацієнта є гормонально активна пухлина кори наднирників. Гіперпродукція якого гормону є причиною вторинної артеріальної гіпертензії у хворого? An examination of a patient with high blood pressure revealed secondary arterial hypertension. It was found that the cause of the patient's condition is a hormonally active tumor of the adrenal cortex. Hyperproduction of which hormone is the cause of secondary arterial hypertension in the patient?

Глюкагон Glucagon

Тироксин Thyroxine

Інсулін Insulin

Кортизол Cortisol

Адреналін Adrenaline

89 / 158
Тривале лiкування гiпофункцiї щитоподiбної залози спричинило загальну дистрофiю, карiєс зубiв, тахiкардiю, тремор кiнцiвок. Який засiб викликав побiчнi ефекти? Long-term treatment of hypothyroidism caused general dystrophy, dental caries, tachycardia, tremors of the limbs. Which drug caused side effects?

Паратиреоїдин Parathyroidin

L-тироксин L-thyroxine

Тирокальцитонiн Tyrocalcitonin

Хумулiн Humulin

Преднiзолон Prednisolone

90 / 158
При обстеженні у алерголога хворому встановлений діагноз - поліноз. У який спосіб можна провести специфічну десенсибілізацію? When examined by an allergist, the patient was diagnosed with hay fever. How can specific desensitization be carried out?

Ілюкокортикоїди Ilucocorticoids

Введення фізіологічного розчину Saline injection

Дробне введенням алергену Minor allergen injection

Атигістамінні препарати Antihistamines

- -

91 / 158
У хірургічному відділенні перев'язувальні матеріали стерилізували в автоклаві. За недоглядом медсестри режим стерилізації був порушений і температура в автоклаві досягла 100оС замість належних 120оС. Які мікроорганізми можуть зберігати життєздатність у таких умовах? In the surgical department, dressing materials were sterilized in an autoclave. Due to the nurse's negligence, the sterilization regime was violated and the temperature in the autoclave reached 100°C instead of the proper 120°C. What microorganisms can remain viable in such conditions?

Коринебактерії та мікобактерії Corynebacteria and mycobacteria

Стафілококи та стрептококи Staphylococci and streptococci

Бацили та клостридії Bacilli and Clostridia

Сальмонели та клебсієли Salmonella and Klebsiella

Плісняві та дріжджеподібні грибки Molds and yeasts

92 / 158
Жінці 26-ти років, хворій на бронхіт, призначили засіб етіотропної терапії антибіотик широкого спектру дії. Який це препарат? A 26-year-old woman with bronchitis was prescribed a broad-spectrum antibiotic as a means of etiotropic therapy. What is this drug?

Амброксол Ambroxol

Дексаметазон Dexamethasone

Інтерферон Interferon

БЦЖ-вакцина BCG vaccine

Доксициклін Doxycycline

93 / 158
У хворого на алкоголізм почався алкогольний психоз із вираженим психомоторним збудженням. Який препарат з групи нейролептиків слід призначити для швидкої допомоги? A patient with alcoholism has started an alcoholic psychosis with pronounced psychomotor agitation. Which drug from the group of neuroleptics should be prescribed for emergency care?

Діазепам Diazepam

Резерпін Reserpin

Фторотан Fluorotan

Аміназин Aminazine

Натрію бромід Sodium bromide

94 / 158
У препараті яєчника поряд з фолікулами різного порядку виявляються атретичні тіла і розвинуте жовте тіло. Якій стадії оваріально-ментруального циклу відповідає такий стан у яєчнику? Atretic bodies and a developed corpus luteum are found in the preparation of the ovary along with follicles of various orders. What stage of the ovarian-menstrual cycle does this state in the ovary correspond to?

Регенераторна Regenerative

Передменструальна Premenstrual

Росту фолікула Follicle Growth

Постменструальна Postmenstrual

Менструальна Menstrual

95 / 158
Пацієнта госпіталізовано з попреднім діагнозом «гепатит В». Для діагностики захворювання зачточовано серологічну реакцію, яка грунтується на взаємодії антигена з антитілом, хімічно зв’язаним з пероксидазою або лужною фосфатазою. Яку назву має використана серологічна реакція? The patient was hospitalized with a previous diagnosis of 'hepatitis B'. To diagnose the disease, a serological reaction was performed, which is based on the interaction of an antigen with an antibody chemically linked to peroxidase or alkaline phosphatase. What is the name of the serological reaction used?

Імуноферментний аналіз Enzyme immunoassay

Реакція імобілізації Immobilization reaction

Реакція імунофлюоресценції Immunofluorescence reaction

Радіоімунологічний метод Radioimmunological method

Реакція зв’язування комплементу Complement binding reaction

96 / 158
Чоловік 63 років, страждає раком стравоходу, метастази в лімфатичні вузли середостіння, ракова кахексія. Яка патогенетична стадія пухлинного процесу чоловіка? A 63-year-old man suffers from esophageal cancer, metastases in the mediastinal lymph nodes, cancer cachexia. What is the pathogenetic stage of the man's tumor process?

прогресії progressions

трансформації transformations

- -

промоції promotions

ініціації initiations

97 / 158
У чоловіка швидкість клубочкової фільтрації 180 мл/хв (норма -125±25 мл/хв).Причиною цього може бути зменшення: A man has a glomerular filtration rate of 180 ml/min (norm -125±25 ml/min). The reason for this may be a decrease:

Онкотичного тиску плазми крові Oncotic blood plasma pressure

Гідростатичного тиску крові в капілярах клубочків Hydrostatic blood pressure in glomerular capillaries

Ниркового кровотоку Renal blood flow

Проникності ниркового фільтру Renal filter permeability

Ефективного фільтраційного тиску Effective filtration pressure

98 / 158
У немовляти присутній мікроцефалія. Лікарі вважають, що це пов'язано із застосуванням жінкою під час вагітності актиноміцину D. На які зародкові листки подіяв цей тератоген? The baby has microcephaly. Doctors believe that this is due to the woman's use of actinomycin D during pregnancy. Which germ layers did this teratogen affect?

Ентодерми і мезодерма Entoderm and mesoderm

Ентодерма Entoderm

Ектодерма Ectoderm

Мезодерма Mesoderm

Усі листки All leaves

99 / 158
Основна маса азоту з організму виводиться у вигляді сечовини. Зниження активності якого ферменту в печінці призводить до гальмування синтезу сечовини і нагромадження амоніаку в крові і тканинах? The main mass of nitrogen is excreted from the body in the form of urea. A decrease in the activity of which enzyme in the liver leads to the inhibition of urea synthesis and the accumulation of ammonia in the blood and tissues?

Карбамоїлфосфатсинтаза Carbamoyl phosphate synthase

Уреаза Urease

Пепсин Pepsin

Аспартатамінотрансфераза Aspartate aminotransferase

Амілаза Amylase

100 / 158
У хворого на артеріальну гіпертензію наслідком гіпертонічного кризу стала гостра серцева недостатність. Який механізм серцевої недостатності є головним в даному випадку? A patient with arterial hypertension developed acute heart failure as a result of a hypertensive crisis. What is the main mechanism of heart failure in this case?

Відносна коронарна недостатність Relative coronary insufficiency

Ушкодження міокарда Myocardial damage

Перевантаження серця опором Heart overload with resistance

Перевантаження серця припливом крові Overload of the heart with blood flow

Абсолютна коронарна недостатність Absolute coronary insufficiency

101 / 158
14. При розтині померлого, який хворів на туберкульоз, у верхній частці правої легені знайдено порожнину розмірами 3x2 см, яка сполучається з бронхом. Стінка порожнини щільна, має три шари: внутрішній -піогенний, середній - шар туберкульозної грануляційної тканини, зовнішній - сполучнотканинний. Який діагноз найбільш імовірний? 14. During the autopsy of the deceased who had tuberculosis, a 3x2 cm cavity was found in the upper part of the right lung, which communicates with the bronchus. The wall of the cavity is dense, has three layers: inner - pyogenic, middle - a layer of tuberculous granulation tissue, outer - connective tissue. What is the most likely diagnosis?

Гострий осередковий туберкульоз Acute focal tuberculosis

Туберкульома Tuberculoma

Гострий кавернозний тубеокульоз Acute cavernous tubeoculosis

Фіброзно-кавернозний туберкульоз Fibro-cavernous tuberculosis

Фіброзно-осередковий туберкульоз Fibrocellular tuberculosis

102 / 158
У жінки 30-ти років з’явилися ознаки вірилізму (ріст волосся на тілі, облисіння скронь, порушення менструального циклу). Гіперпродукція якого гормону може спричинити такий стан? A 30-year-old woman has signs of virilism (growth of body hair, balding of the temples, irregular menstrual cycle). Hyperproduction of which hormone can cause such a condition?

Релаксин Relaxin

Пролактин Prolactin

Окситоцин Oxytocin

Естріол Estriol

Тестостерон Testosterone

103 / 158
У хворого хлопчика 12-ти років вміст холестерину в сироватці крові до 25 ммоль/л. В анамнезі - спадкова сімейна гіперхолестеринемія, причиною якої є порушення синтезу білків-рецепторів до: A sick 12-year-old boy has serum cholesterol up to 25 mmol/l. He has a history of hereditary familial hypercholesterolemia, the cause of which is a violation of the synthesis of receptor proteins to:

Ліпопротеїнів високої щільності High-density lipoprotein

Хіломікронів Chylomicrons

Ліпопротеїнів низької щільності Low-density lipoprotein

Ліпопротеїнів проміжної щільності Intermediate density lipoprotein

Ліпопротеїнів дуже низької щільності Very low density lipoprotein

104 / 158
Під дією негативних чинників довкілля порушена функція міосателітоцитів. Зміну якої функції всього м’язового волокна слід очікувати в даному випадку? Under the influence of negative environmental factors, the function of myosatellitocytes is disturbed. What change in the function of the entire muscle fiber should be expected in this case?

Регенерація Regeneration

Скоротливий термогенез Reductive thermogenesis

- -

Трофіка Trofics

Скорочення Abbreviation

105 / 158
Чоловік внаслідок ДТП отримав травму і втратив велику кількість крові. Які зміни периферичної крові будуть найбільш імовірні на 2-й день після травми? A man was injured in a road accident and lost a large amount of blood. What changes in peripheral blood are most likely to occur on the 2nd day after the injury?

Анізоцитоз Anisocytosis

Значний ретикулоцитоз Significant reticulocytosis

Гіпохромія Hypochromia

Еритропенія Erythropenia

Пойкілоцитоз Poikilocytosis

106 / 158
Типовими проявами харчового отруєння, спричиненого С.ЬоІиІіпит є двоїння в очах, порушення ковтання та дихання. Ці симптоми розвиваються внаслідок: Typical manifestations of food poisoning caused by S. lypiitis are double vision, difficulty swallowing and breathing. These symptoms develop as a result of:

Дії екзотоксину Effects of Exotoxin

Активації аденілатциклази Adenylate cyclase activations

Дії ентеротоксину Effects of enterotoxin

Адгезії збудника до рецепторів на ентероцитах Adhesions of pathogen to receptors on enterocytes

Розвитку ентеротоксичного шоку Development of enterotoxic shock

107 / 158
Підліток 15 років, скаржиться на недостачу повітря, загальну слабкість, серцебиття. ЧСС 130/хв, АТ-100/60 мм рт. ст. на ЕКГ, комплекс QRS нормальної форми та тривалості. Число зубців Р та шлункових комплексів однакове, зубець Т злитий з зубцем Р. Яка аритмія серця спостерігається у підлітка? 15-year-old teenager, complains of lack of air, general weakness, palpitations. Heart rate 130/min, blood pressure 100/60 mm Hg on ECG, complex QRS of normal shape and duration. The number of P waves and gastric complexes are the same, the T wave is merged with the P wave. What cardiac arrhythmia is observed in a teenager?

Тремтіння передсердь Atrial fibrillation

Синусова екстрасистолія Sinus extrasystole

Синусова тахікардія Sinus tachycardia

Мерехтіння передсердь Atrial fibrillation

Передсердна пароксизмальна тахікардія Atrial paroxysmal tachycardia

108 / 158
У хворого після інфаркту міокарда передсердя і шлуночки стали скорочуватись незалежно одне від одного: з частотою 60-70 та 35-40 за хвилину. Назвіть вид блокади в даному випадку: After a myocardial infarction, the patient's atria and ventricles began to contract independently of each other: with a frequency of 60-70 and 35-40 per minute. Name the type of blockade in this case :

Повна передсердно-шлуночкова Complete atrioventricular

Сино-атріальна Sino-atrial

Внутрішньопередсердна Intraatrial

Неповна передсердно-шлуночкова Incomplete atrioventricular

Внутрішньошлуночкова Intraventricular

109 / 158
Коензим А бере участь в багатьох важливих реакціях метаболізму. Похідним якого вітаміну він є? Coenzyme A is involved in many important metabolic reactions. What vitamin is it a derivative of?

Тіамін Thiamine

Кальциферол Calciferol

Пантотенова кислота Pantothenic acid

Убіхінон Ubiquinone

Ніацин Niacin

110 / 158
Жінці 46-ти років планується провести об’ємне оперативне втручання в щелепно-лицьовій області. Відомо, що хвора схильна до підвищеної гемокоа-гуляції. Який природний антикоагулянт може бути застосований з метою запобігання тромбоутворенню? A 46-year-old woman is scheduled to undergo extensive surgical intervention in the maxillofacial region. It is known that the patient is prone to increased hemocoagulation. What natural anticoagulant can be used to prevent blood clot formation?

Фібринолізин Fibrinolysin

Цитрат натрію Sodium Citrate

Гірудин Hirudin

Жодна з наведених речовин None of these substances

Гепарин Heparin

111 / 158
Хворий у непритомному стані доставлений бригадою швидкої допомоги у лікарню. Об'єктивно: рефлекси відсутні, періодично з’являються судоми, дихання нерівномірне. Після лабораторного обстеження було діагностовано печінкову кому. Нагромадження якого метаболіту є суттєвим для появи розладів центральної нервової системи? The patient was brought to the hospital in an unconscious state by an ambulance. Objectively: reflexes are absent, convulsions appear periodically, breathing is irregular. After a laboratory examination, liver disease was diagnosed to whom. The accumulation of which metabolite is essential for the appearance of disorders of the central nervous system?

Амоніак Ammonia

Сечовина Urea

Білірубін Bilirubin

Глутамін Glutamine

Гістамін Histamine

112 / 158
Під час внутрішньовенної трансфузії фізіологічним розчином стан хворого різко погіршився і на фоні гострої ядухи настала смерть. При розтині померлого виявлено гостре венозне повнокров'я внутрішніх органів з різко розширеними правими відділами серця. При проколі правого шлуночка під водою виявляються пухирці. Який патологічний процес та його вид виник у хворого? During the intravenous transfusion with physiological solution, the patient's condition worsened sharply and death occurred against the background of acute dysentery. At the autopsy of the deceased, acute venous congestion of the internal organs with sharply enlarged right departments of the heart. When the right ventricle is punctured, bubbles are found under water. What pathological process and its type occurred in the patient?

Жирова емболія Fat embolism

Тканинна емболія Tissue embolism

Повітряна емболія Air embolism

Тромбоемболія Thromboembolism

ГІазова емболія GIas embolism

113 / 158
При первинному посіві води на 1% пе-птонну воду, через 6 годин на поверхні середовища виявлений ріст - ніжна плівка. Для збудника якого захворювання характерні такі культуральні властивості? During the initial inoculation of water with 1% peptone water, after 6 hours growth was detected on the surface of the medium - a delicate film. What disease pathogen is characterized by such cultural properties?'

Дизентерія Dysentery

Холера Cholera

Туберкульоз Tuberculosis

Псевдотуберкульоз Pseudotuberculosis

Чума Plague

114 / 158
Робітник одного із сільськогосподарських підприємств гостро занедужав і при наростаючих явищах інтоксикації помер. На розтині тіла: селезінка збільшена, в'яла, на розрізі - темно-вишневого кольору, дає надмірний зішкріб пульпи. М’які мозкові оболонки на склепінні та основі мозку набряклі, просякнуті кров'ю ('червоний чепець” або 'шапочка кардинала'). Мікроскопічно: серозно-геморагічне запалення оболонок і тканин мозку. Який найбільш імовірний діагноз? A worker at one of the agricultural enterprises became acutely ill and died due to increasing symptoms of intoxication. On autopsy: the spleen was enlarged, flaccid, on cross-section it was dark cherry-colored, gives excessive scraping of the pulp. The soft meninges on the vault and base of the brain are swollen, soaked with blood ('red cap' or 'cardinal cap'). Microscopically: serous-hemorrhagic inflammation of the meninges and tissues of the brain. What is the most likely diagnosis?

Туляремія Tularemia

Сибірка Anthrax

Чума Plague

Холера Cholera

Бруцельоз Brucellosis

115 / 158
В клініку поступила молода жінка зі скаргами на різкий біль внизу живота. При обстеженні у лікаря виникла підозра на розрив маткової труби в результаті позаматкової вагітності. Під час пункції через заднє склепіння піхви шприці виявили кров. В якому з утво-рень очеревини жінки накопичується кров в даному випадку? A young woman was admitted to the clinic with complaints of sharp pain in the lower abdomen. During the examination, the doctor suspected a fallopian tube rupture as a result of an ectopic pregnancy. During a puncture through the posterior the vault of the vagina, the syringe revealed blood. In which of the formations of the woman's peritoneum does the blood accumulate in this case?

Міхурно-матковому поглибленні. To vesical-uterine deepening.

Прямокишково-матковому поглибленні. Recto-uterine deepening.

Міжсигмовидному поглибленні. Intersigmoid deepening.

Прямокишково-міхурному поглибленні. Rectovesical deepening.

Позадусліпокишковому поглибленні. Postcecal indentation.

116 / 158
Чоловік 42 років страждає ревматоїдним артритом До комплексу призначених йому лікувальних препаратів включений аспірин - інгібітор. З якої кислоти утворюються простагландини? A 42-year-old man suffers from rheumatoid arthritis. Aspirin, an inhibitor, is included in the complex of drugs prescribed for him. What acid is used to form prostaglandins?

Нейрамінової Neuraminova

Пропіонової Propionova

Арахідоново Arachidonovo

Ліноленової Linolenova

Лінолевої Linolev

117 / 158
У вагітної жінки акушер-гінеколог вимірює розміри таза. За допомогою циркуля була виміряна відстань між двома клубовими гребенями. Який розмір таза був визначений? An obstetrician-gynecologist measures the size of a pregnant woman's pelvis. The distance between the two iliac crests was measured using a compass. What size of the pelvis was determined?

Distantia spinarum Distantia spinarum

Distantia cristarum Distantia cristarum

Distantia throchanterica Distantia throchanterica

Conjugata vera Conjugata vera

Conjugata anatomica Conjugata anatomicala

118 / 158
Під час гістологічного дослідження стулок мітрального клапана серця жінки 30-ти років було встановлено, що ендотеліальні клітини вогнищево десквамовані, в цих ділянках на поверхні стулки розташовані дрібні тромботичні нашарування, сполучна тканина стулки з явищами мукоїдного набухання з ділянками склерозу та васкуляризації. Діагностуйте вид клапанного ендокардиту: During a histological examination of the leaflets of the mitral valve heart of a 30-year-old woman, it was established that the endothelial cells are focally desquamated, in these areas there are small thrombotic layers on the surface of the leaflet, leaflet connective tissue with mucoid swelling phenomena with areas of sclerosis and vascularization. Diagnose the type of valvular endocarditis:

Дифузний Diffuse

Поворотньо-бородавчастий Rotary-warty

Поліпозно-виразковий Polypo-ulcerative

Гострий бородавчастий Sharp warty

Фібропластичний Fibroplastic

119 / 158
До лікарні доставлено жінку з симптомами гострого апендициту, що супроводжується напруженням м'язів у правій здухвинній ділянці. Який тип вегетативних рефлексів забезпечує виникнення даного симптому? A woman was brought to the hospital with symptoms of acute appendicitis accompanied by muscle tension in the right thoracic region. What type of autonomic reflexes causes this symptom?

Дермато-вісцеральні Dermato-visceral

Вісцеро-соматичні Viscero-somatic

Вісцеро-вісцеральні Viscero-visceral

Сомато-вісцеральні Somato-visceral

Вісцеро-дермальні Viscero-dermal

120 / 158
Мембранний потенціал спокою клітини змінився з -85 до -90 мВ. Причиною цього може бути активація таких каналів мембрани клітини: The resting membrane potential of the cell has changed from -85 to -90 mV. The reason for this may be the activation of the following channels of the cell membrane:

Калієві та натрієві Potassium and sodium

Калієві та кальцієві Potassium and calcium

Кальцієві Calcium

Натрієві Sodium

Калієві Potassium

121 / 158
До щелепно-лицьового відділення надійшов хворий з переломом нижньої щелепи. Було вирішено з'єднання кісток провести хірургічним методом під наркозом. Після внутрішньовенного введення міорелаксанту спостерігались короткочасні фібрилярні скорочення м'язів обличчя хворого. Який міорелаксант було застосовано? A patient with a fracture of the lower jaw was admitted to the maxillofacial department. It was decided to join the bones surgically under anesthesia. After intravenous administration of a muscle relaxant, short-term fibrillar contractions were observed 'muscles of the patient's face. What muscle relaxant was used?

Дитилін Ditylin

Тубокурарина хлорид Tubocurarine chloride

Меліктин Melictin

Пипекуроній бромід Pipecuronium bromide

Діазепам Diazepam

122 / 158
У жінки 35-ти років проведено біопсію вузлових утворень молочної залози. В ході гістологічного дослідження відзначається посилена проліферація епітелію дрібних протоків та ацинусів з формуванням залозистоподібних структур, різної форми та розмірів, які розташовані у фіброзній стромі. У ділянках залозистої проліферації ознак клітинної атипії не виявлено. Який найбільш імовірний діагноз? A 35-year-old woman underwent a biopsy of nodular formations of the mammary gland. During histological examination, increased proliferation of the epithelium of small ducts and acini with the formation of gland-like structures of various shapes and sizes that are located in the fibrous stroma. No signs of cellular atypia were found in the areas of glandular proliferation. What is the most likely diagnosis?

Кістозна мастопатія Cystic mastopathy

Мастит Mastitis

Аденокарцинома Adenocarcinoma

Інвазивна протокова карцинома Invasive ductal carcinoma

Фіброаденома Fibroadenoma

123 / 158
У неврологічному відділенні з приводу оперізуючого лишаю проходить курс лікування жінка 50-ти років. Внаслідок реактивації якого вірусу виникло це захворювання? A 50-year-old woman is being treated for shingles in the neurological department. What virus caused this disease?

Вірусу простого ГЕРПЕСУ 1-го типу Herpes simplex virus type 1

Вірусу кору Measles virus

Вірусу простого ГЕРПЕСУ 2-го типу Herpes simplex virus type 2

Вірусу Зостер (вітряної віспи) Zoster virus (chicken pox)

Вірусу цитомегалії Cytomegaly virus

124 / 158
У пацієнта 28-ми років, який знаходиться в пульмонологічному відділенні, діагностовано емфізему легенів, яка виникла внаслідок розщеплення міжальвеолярних перетинок тканинним трипсином. Уроджена недостатність якого білка спричинила розвиток захворювання? A 28-year-old patient, who is in the pulmonology department, was diagnosed with emphysema of the lungs, which arose as a result of the splitting of the interalveolar membranes by tissue trypsin. Congenital deficiency of which protein caused the development of the disease ?

Кріоглобулін Cryoglobulin

аі-протеїназний інгібітор AI proteinase inhibitor

Гаптоглобін Haptoglobin

а2-макроглобулін a2-macroglobulin

Трансферин Transferin

125 / 158
Хворому на гострий інфаркт міокарда у комплексній терапії було призначено гепарин. Через деякий час після введення даного препарату з’явилася гематурія. Який антагоніст гепарину необхідно ввести хворому для усунення даного ускладнення? A patient with an acute myocardial infarction was prescribed heparin in complex therapy. Some time after the administration of this drug, hematuria appeared. What heparin antagonist should be administered to the patient to eliminate this complication?

Амінокапронова кислота Aminocaproic acid

Неодикумарин Neodicoumarin

Вікасол Vikasol

Фібриноген Fibrinogen

Протаміну сульфат Protamine sulfate

126 / 158
У хворого, який проходить курс лікувального голодування, нормальний рівень глюкози у крові підтримується головним чином за рахунок глюконеогенезу. З якої амінокислоти при цьому у печінці людини найбільш активно синтезується глюкоза? In a patient who undergoes a course of therapeutic fasting, the normal level of glucose in the blood is maintained mainly due to gluconeogenesis. From which amino acid is the most actively synthesized glucose in the human liver ?

Лізин Lysine

Глутамінова кислота Glutamic acid

Лейцин Leucine

Аланін Alanine

Валін Valin

127 / 158
Хворому для зупинки кровотечі призначили препарат - прямий коагулянт. При введенні розчину пацієнт скаржився на біль по ходу вени, відчуття жару, серцебиття. Назвіть препарат, який викликає такі симптоми: The patient was prescribed a drug to stop bleeding - a direct coagulant. When the solution was administered, the patient complained of pain along the course of the vein, a feeling of heat, palpitations. Name the drug that causes these symptoms :

Ергокальциферол Ergocalciferol

Стрептокіназа Streptokinase

Кальцію хлорид Calcium chloride

Гірудин Hirudin

Пентоксил Pentoxyl

128 / 158
Які рецептори слід заблокувати у людини перед проведенням бронхоскопії, щоб зменшити вплив блукаючого нерва на гладкі м’язи бронхів? Which receptors should be blocked in a person before bronchoscopy to reduce the influence of the vagus nerve on the smooth muscles of the bronchi?

М-холінорецептори M-cholinergic receptors

в-адренорецептори v-adrenoceptors

а- і в-адренорецептори a- and b-adrenoceptors

Н-холінорецептори H-cholinergic receptors

а-адренорецептори α-adrenoceptors

129 / 158
У працівника хімчистки виявлена жирова дистрофія печінки. Порушення синтезу якої речовини може призвести до даної патології? A dry cleaner's worker was diagnosed with fatty liver dystrophy. Violation of the synthesis of which substance can lead to this pathology?

Холева кислота Cholic acid

Сечовина Urea

Тристеарин Tristearin

Фосфатидилхолін Phosphatidylcholine

Фосфатидна кислота Phosphatic acid

130 / 158
У хворого виявлені спленомегалія, геморагічний діатез. Периферична кров не змінена. При пункції груднини: кістковий мозок представлений тільки мієлобластами. Про яке захворювання йдеться? The patient has splenomegaly, hemorrhagic diathesis. Peripheral blood has not changed. At sternal puncture: the bone marrow is represented only by myeloblasts. What disease is it about?

Мієлоцитарний лейкоз Myelocytic leukemia

Лімфоцитарний лейкоз Lymphocytic leukemia

Мієлобластний лейкоз Myeloblastic leukemia

Лімфобластний лейкоз Lymphoblastic leukemia

Лімфосаркома Lymphosarcoma

131 / 158
У людей, адаптованих до дії високої зовнішньої температури, посилене потовиділення не супроводжується втратою з потом великої кількості хлориду натрію. Дія якого гормону на потові залози спричинює цей результат? In people adapted to the action of high external temperatures, increased sweating is not accompanied by the loss of large amounts of sodium chloride in sweat. What hormone's action on the sweat glands causes this result?

Альдостерон Aldosterone

Натрійуретичний Natriuretic

Вазопресин Vasopressin

Тироксин Thyroxine

0% Кортизол 0% Cortisol

132 / 158
В відділення черепно-мозкової травми потрапив хворий з пошкодженням великого крила клиноподібної кістки. Лінія перелому пройшла через ОСТИСТИЙ отвір. Яка судина постраждала? A patient with damage to the large wing of the sphenoid bone was admitted to the brain injury department. The fracture line passed through the SPINAL foramen. Which vessel was injured?

Передня глибока скронева артерія Anterior deep temporal artery

Латеральна крилоподібна артерія Lateral pterygoid artery

Середня оболонна артерія Medium meningeal artery

Задня глибока скронева артерія Posterior deep temporal artery

Поверхнева скронева артерія Superficial temporal artery

133 / 158
Хворому для зупинки кровотечі призначили препарат - прямий коагулянт. При введенні розчину пацієнт скаржився на біль по ходу вени, відчуття жару, серцебиття. Назвіть препарат, який викликає такі симптоми: The patient was prescribed a drug to stop bleeding - a direct coagulant. When the solution was administered, the patient complained of pain along the course of the vein, a feeling of heat, palpitations. Name the drug that causes these symptoms :

Кальцію хлорид Calcium chloride

Стрептокіназа Streptokinase

Гірудин Hirudin

Ергокальциферол Ergocalciferol

Пентоксил Pentoxyl

134 / 158
Жінці 23-х років в комплексному лікування ксерофтальмії лікар призначив ретинолу ацетат, але, дізнавшись, що пацієнтка знаходиться на 8-му тижні вагітності, відмінив зазначений лікарський засіб. Яка можлива дія віта-мінопрепарату спонукала лікаря переглянути призначення? The doctor prescribed retinol acetate to a 23-year-old woman in the complex treatment of xerophthalmia, but after learning that the patient is in the 8th week of pregnancy, he canceled the indicated medicine. What possible action of the vitamin mino drug prompted the doctor to reconsider the appointment?

Тератогенна Teratogenic

Токсична Toxic

Мутагенна Mutagenic

Утеротонічна Uterotonic

Канцерогенна Carcinogenic

135 / 158
Надмірна секреція певного гормону спостерігалася у хворого при феохромоцитомі - пухлині, що походить з мозкової речовини надниркових залоз. Як він називається? Excessive secretion of a certain hormone was observed in a patient with pheochromocytoma - a tumor originating from the medulla of the adrenal glands. What is it called?

Глюкагон Glucagon

Тироксин Thyroxine

Адреналін Adrenaline

Соматотропін Somatotropin

Інсулін Insulin

136 / 158
У пацієнта 36-ти років після дорожньої травми виникли параліч м’язів кінцівок справа, втрата больової і температурної чутливості зліва, часткове зниження тактильної чутливості з обох сторін. Для ураження якого відділу мозку вказані зміни є найбільш характерними? A 36-year-old patient developed paralysis of the muscles of the limbs on the right, loss of pain and temperature sensitivity on the left, and a partial decrease in tactile sensitivity on both sides after a traffic injury. damage to which part of the brain are the specified changes most characteristic?

Передні стовпи спинного мозку Anterior columns of the spinal cord

Рухова кора зліва Left motor cortex

Ліва половина спинного мозку Left half of spinal cord

Задні стовпи спинного мозку Posterior columns of the spinal cord

Права половина спинного мозку Right half of spinal cord

137 / 158
Після ін’єкції інсуліну рівень глюкози крові знижується протягом декількох секунд. Це відбувається завдяки активації такого процесу: After an insulin injection, the blood glucose level decreases within a few seconds. This is due to the activation of the following process:

Синтез глікогену Glycogen synthesis

Пентозофосфатний цикл Pentose phosphate cycle

Синтез ліпідів Synthesis of lipids

Транспорт глюкози до клітин Transport of glucose to cells

Гліколіз Glycolysis

138 / 158
В ході біохімічного аналізу еритроцитів немовляти встановлено виражену недостатність глутатіонпероксидази і низький рівень відновленого глутатіону. Який патологічний стан може розвинутися у цієї дитини? During the biochemical analysis of the baby's erythrocytes, a pronounced deficiency of glutathione peroxidase and a low level of reduced glutathione were established. What pathological condition can develop in this child?

Гемолітична анемія Hemolytic anemia

Мегалобластна анемія Megaloblastic anemia

Перніціозна анемія Pernicious anemia

Залізодефіцитна анемія Iron deficiency anemia

Серпоподібно-клітинна анемія Sickle cell anemia

139 / 158
Жінка 30-ти років хворіє близько року, коли вперше з'явився біль у ділянці суглобів, їх припухлість, почервоніння шкіри над ними. Попередній діагноз - ревматоїдний артрит. Зміна якого компоненту в структурі білка сполучної тканини є однією з причин цього захворювання? A 30-year-old woman has been sick for about a year, when pain in the area of the joints, their swelling, redness of the skin above them first appeared. The preliminary diagnosis is rheumatoid arthritis. A change in which component in the protein structure of the connective tissue is one of the causes of this disease?

Овоальбумін Ovoalbumin

Міозин Myosin

Колаген Collagen

Муцин Mucin

Тропонін Troponin

140 / 158
При розтині померлого, який хворів на туберкульоз, у верхній частці правої легені знайдено порожнину розмірами 3x2 см, яка сполучається з бронхом. Стінка порожнини щільна, має три шари: внутрішній -піогенний, середній - шар туберкульозної грануляційної тканини, зовнішній - сполучнотканинний. Який діагноз найбільш імовірний? During the autopsy of the deceased who had tuberculosis, a 3x2 cm cavity was found in the upper part of the right lung, which connects to the bronchus. The wall of the cavity is dense and has three layers: inner - pyogenic, middle - layer of tuberculous granulation tissue, outer - connective tissue. What is the most likely diagnosis?

Гострий осередковий туберкульоз Acute focal tuberculosis

Гострий кавернозний тубеокульоз Acute cavernous tubeoculosis

Фіброзно-осередковий туберкульоз Fibrocellular tuberculosis

Фіброзно-кавернозний туберкульоз Fibro-cavernous tuberculosis

Туберкульома Tuberculoma

141 / 158
Під час розтину тіла померлого в черевній порожнині виявлено близько 2,0 л гнійної рідини. Очеревина тьмяна, з сіруватим відтінком, на серозній оболонці кишок є сіруватого кольору нашарування, що легко знімаються. Найімовірніше у хворого був такий перитоніт: During the autopsy of the body of the deceased, about 2.0 liters of purulent fluid was found in the abdominal cavity. The peritoneum is dull, with a grayish tint, on the serous membrane of the intestines there is grayish layering, which are easily removed. Most likely, the patient had such peritonitis:

Туберкульозний Tuberculosis

Серозний Serious

Геморагічний Hemorrhagic

Фібринозно-гнійний Fibrinous-purulent

- -

142 / 158
У пацієнта на ЕКГ спостерігається значне розширення зубця R (до 0,18 с). Це зумовлено зменшенням швидкості проведення збудження такими структурами серця: The patient's ECG shows a significant expansion of the R wave (up to 0.18 s). This is due to a decrease in the speed of conduction of excitation by the following structures of the heart:

Атріо-вентрикулярний вузол Atrio-ventricular node

Лівий шлуночок Left Ventricle

Шлуночки Ventricles

Правий шлуночок Right ventricle

Передсердя Atrial

143 / 158
A patient has peptic ulcer of the stomach. What medicine can decrease the secretion of hydrochloric acid and pepsin by blocking the H2 receptors? У пацієнта виразкова хвороба шлунка. Який препарат може зменшити секрецію соляної кислоти та пепсину шляхом блокування Н2-рецепторів?

Famotidine Фамотидин

Fluvoxamine Флувоксамін

Phthalazol (Phthalylsulfathiazole) Фталазол (фталілсульфатіазол)

Phenobarbital Фенобарбітал

Physostigmine Фізостигмін

144 / 158
Roentgenological examination of skull base bones revealed enlargement of sellar cavity, thinning of anterior clinoid processes, destruction of different parts, destruction of different parts of sella turcica. Such bone destruction might be caused by a tumour of the following wndocrinous gland: Рентгенологічне дослідження кісток основи черепа виявило розширення порожнини селярної кістки, потоншення передніх кліноподібних відростків, деструкцію різних частин, деструкцію різних частин турецького седла. Така кістка деструкція може бути викликана пухлиною такої залози секреції:

Hypophysis Гіпофіз

Thyroid gland Щитовидна залоза

Adrenal glands Надниркові залози

Epiphysis Епіфіз

Thymus gland Вилочкова залоза

145 / 158
A veterinarian, working at cattle farm, visited doctor and complained of pain in joints, fever, weakness, night sweats. He has been ill for a month. According to complaints and patient profession doctor suspected brucellosis. Which material collected from this patient should be exameneed in ordinary bacteriological laboratory? Ветеринар, який працює на тваринницькій фермі, звернувся до лікаря зі скаргами на біль у суглобах, жар, слабкість, нічну пітливість. Хворіє вже місяць. до скарг і професії хворого лікар запідозрив бруцельоз. Який матеріал, зібраний у цього хворого, необхідно дослідити в звичайній бактеріологічній лабораторії?

Vomiting mass Блювотна маса

Feces Кал

Urine Сеча

Cerebrospinal fluid Спинномозкова рідина

Blood serum Сироватка крові

146 / 158
A bone marrow biopsy of an 8-year old girl shows a group of cells which has undergone the process ofpyknosis and loss of the nucleus dunng iis differentiation. Which of the following types of hemopoiesis is characterised by the morphological changes described above? Біопсія кісткового мозку 8-річної дівчинки показує групу клітин, які зазнали процесу пікнозу та втрати ядра через диференціацію. Яка з для наступних типів кровотворення характерні описані вище морфологічні зміни?

Thrombocylopoesis Тромбоцилопоез

Monocytopoesis Моноцитопоез

Lymphocytopoesis Лімфоцитопоез

Erytropoesis Еритропоез

Granulocytopoesis Гранулоцитопоез

147 / 158
A 46-year-old man presents with fatigue and joint pain in his fingers and wrists for the last 2 months. The pain is present in both hands and the wrists are swollen. Furthermore, he describes morning stiffness in his joints lasting about 2 hours, which improves with use. His past medical history reveals he has been successfully treated for II. pylori related ulcers last year. He denies smoking and stopped drinking when his gastric symptoms started. Which of the following drugs is the best choice for his joint pain management? 46-річний чоловік відчуває втому та біль у суглобах у пальцях і зап’ястях протягом останніх 2 місяців. Біль присутній в обох руках і зап'ястя опухлі. Крім того, він описує ранкову скутість у суглобах, яка триває приблизно 2 години, яка покращується під час використання. Його минула медична історія показує, що минулого року він успішно лікувався від виразки, пов'язаної з II.pylori. Він заперечує куріння та кинув пити, коли його почалися шлункові симптоми. Який із наведених нижче препаратів є найкращим вибором для лікування болю в суглобах?

Paracetamol Парацетамол

Celecoxib Целекоксиб

Morphine Морфін

Prednisone Преднізон

Aspirin Аспірин

148 / 158
A 16-year-old girl concerned about her sexual development comes to the physician. She mentions that she has still not had a menstrual period. However, she is otherwise a healthy girl with no significant medical problems since birth. On physical examination, her vital signs are stable. She does not have pubic hair and her breast is slightly elevated with areola remaining in contour with surrounding breast. Which of the following is the most likely cause of this abnormal physical development? 16-річна дівчина, стурбована своїм статевим розвитком, приходить до лікаря. Вона згадує, що у неї досі не було менструації. Однак вона в іншому випадку здорова дівчинка без значних медичних проблем з народження. Під час фізичного обстеження її життєво важливі показники стабільні. У неї немає волосся на лобку, її груди трохи підняті, ареола залишається на контурі навколишньої грудей. Що з наведеного нижче найбільше ймовірна причина цього ненормального фізичного розвитку?

Pancreatic islet insufficiency Недостатність острівців підшлункової залози

Adrenal medulla hyperfunction Гіперфункція мозкової речовини надниркових залоз

Hyperthyroidism Гіпертиреоз

Hypothyroidism Гіпотиреоз

Ovarian insufficiency Недостатність яєчників

149 / 158
An 18-year-old girl comes to her physician with concern about her health because she has not achieved menarche. She denies any significant weight loss, changes in mood, or changes in her appetite. She mentions that her mother told her about mild birth defects, but she cannot recall the specifics. Past medical history and familyhistory are benign. On physical examination, the patient is short in stature, has a short and webbed neck and wide chest. Staining of buccal smear reveals absence of Barr bodies in the nucleus of epithelial cells. A urine pregnancy test is negative. Which of the following genetic disorders is the most likely cause of this patient's condition? 18-річна дівчина звернулася до свого лікаря із занепокоєнням щодо свого здоров’я, оскільки у неї не настала менархе. Вона заперечує будь-яку значну втрату ваги, зміни настрою , або зміни апетиту. Вона згадує, що мати розповідала їй про легкі вроджені вади, але вона не може пригадати подробиць. Історія хвороби та сімейний анамнез є доброякісними. Під час медичного обстеження пацієнтка невисокого зросту, має низький і перетинчастий шия та широка грудна клітка. Фарбування букального мазка виявляє відсутність тілець Барра в ядрі епітеліальних клітин. Тест на вагітність сечі негативний. Яке з перерахованих генетичних захворювань є найбільш імовірною причиною стану даної пацієнтки?

Patau syndrome Синдром Патау

Cri du chat ('cat-cry') syndrome Синдром крику кота

Klinefelter syndrome Синдром Клайнфельтера

Edwards syndrome Синдром Едвардса

Turner syndrome Синдром Тернера

150 / 158
A 14-year old girl presents to the emergency department for evaluation of an 'infected lell'. She states there is no history of trauma but mentions she had a history of sickle cell disease. On physical examination, her upper part of right shin is very painful, red. swollen and hot. Her temperature is 39.2°C. An X-ray shows focal bony lysis and loss of trabecular architecture in the metaphysis of right tibia. Increased activity of which of the following cells is the most likely cause of bone reabsorption in this patient? 14-річна дівчина звертається до відділення невідкладної допомоги для оцінки «інфікованого леля». Вона стверджує, що в анамнезі немає травми, але згадує, що мала Серповидно-клітинна анемія в анамнезі. При фізикальному огляді її верхня частина правої гомілки дуже болюча, червона, набрякла та гаряча. Її температура 39,2°C. Рентгенівський знімок показує вогнищевий лізис кістки та втрату трабекулярної архітектури в метафізі правої великогомілкової кістки Підвищена активність якої з наступних клітин є найбільш вірогідною причиною реабсорбції кісткової тканини у цього пацієнта?

Osteocytes Остеоцити

Chondroblasts Хондробласти

Chondrocytes Хондроцити

Osteoblasts Остеобласти

Osteoclasts Остеокласти

151 / 158
A 24-year-old man undergoes surgery and during the operation, an organ is excised and sent for histological evaluation. A light microscopic examination reveals the organ encased by thin connective tissue capsule that enters the substance of the lobes to further subdivide the organ into irregular lobular units. Each lobule contains a cluster of follicles filled with colloid. Follicular epithelium consists of low columnar, cuboidal or squamous cells depending on the level of activity of the follicle. Which of the following organs does this tissue most likely belong to? 24-річному чоловікові роблять операцію, і під час операції орган вирізають і відправляють на гістологічне дослідження. Світлове мікроскопічне дослідження виявляє орган, оточений тонка сполучнотканинна капсула, яка входить у речовину часточок, щоб далі поділити орган на неправильні часточкові одиниці. Кожна часточка містить скупчення фолікулів, заповнених колоїдом. Фолікулярний епітелій складається з низьких стовпчастих, кубоподібних або плоских клітин залежно від рівня активності фолікул. До якого з перерахованих органів найімовірніше належить ця тканина?

Thyroid gland Щитовидна залоза

Pancreas Підшлункова залоза

Thymus Тимус

Parotid gland Привушна залоза

Parathyroid gland Паращитовидна залоза

152 / 158
A 6-year-old boy is brought to the pediatrician by his mother, who complains of low-grade fever, chronic cough and night sweats in her child. She describes the cough as productive, producing white sputum that is sometimes streaked with blood. She also says that her son has lost some weight in the last month. His vital signs include blood pressure of 115/75 mm Hg, heart rate of 110/min., respiratory rate of 18/min. and temperature of 36.6oC. On physical examination, the patient is ill looking. Pulmonary auscultation reveals some fine crackles in the right upper lobe. The pediatrician suspects an active infection and performs Mantoux test. Intradermal injection of which of the following substances has been most likely used by pediatrician for screening test in this clinical case? Мати привела хлопчика 6 років до педіатра, яка скаржиться на субфебрильну температуру, хронічний кашель і нічну пітливість. Вона описує кашель як продуктивний, з виділенням білого мокротиння, іноді з прожилками крові.Вона також каже, що її син трохи схуд за останній місяць.Його життєві показники включають артеріальний тиск 115/75 мм рт.ст., пульс 110/. хв., частота дихання 18/хв., температура 36,6oC.Під час фізикального огляду пацієнт погано виглядає.Під час аускультації легенів у правій верхній частці виявляються дрібні хрипи.Педіатр підозрює активну інфекцію та проводить пробу Манту.Внутрішньошкірно ін'єкція якої з наведених нижче речовин, швидше за все, була використана педіатром для скринінгового тесту в цьому клінічному випадку?

- -

Bacillus Calmette-Guerin (BCG) vaccine Вакцина проти бацили Кальмета-Герена (БЦЖ)

Diphtheria-tetanus toxoids-acellular pertussis vaccine (DTaP) Дифтерійно-правцевий анатоксини безклітинна кашлюкова вакцина (DTaP)

Tetanus and diphtheria toxoids vaccine(Td) Вакцина проти правцевого та дифтерійного анатоксинів (Td)

Tuberculin Туберкулін

153 / 158
A team of medical students is performing research on phases of cell cycle. During one of the mitotic phases the cell is nearly done dividing, the chromosomes décondensé and two nuclei begin to form around them. Which of the following phases most likely takes place in the cell? Команда студентів-медиків проводить дослідження фаз клітинного циклу. Під час однієї з мітотичних фаз клітина майже завершує поділ, хромосоми розпадаються і два ядра навколо них починають формуватися. Яка з наведених фаз найімовірніше протікає в клітині?

- -

Anaphase Анафаза

Metaphase Метафаза

Prophase Профаза

Telophase Телофаза

154 / 158
A 60-year old man with a history of hypertension, diabetes and hyperlipidemia had a sudden onset of right-sided weakness. By the time the ambulance arrived, he had difficulty speaking. Unfortunately, the patient died within the next 2 hours and an autopsy was performed immediately. The gross examination of the cerebral left hemisphere showed brain swelling, widened gyri and poorly demarcated gray-white junction. Which of the following is the most likely cause of this patient's death? У 60-річного чоловіка з гіпертонією, діабетом і гіперліпідемією раптово виникла слабкість правого боку. Коли приїхала швидка допомога, він мав проблеми з розмовою. На жаль, пацієнт помер протягом наступних 2 годин, і негайно було проведено розтин. Загальний огляд лівої півкулі головного мозку показав набряк мозку, розширені звивини та погано розмежований сіро-білий з’єднання. Що з перерахованого найбільше ймовірна причина смерті цього пацієнта?

Intracerebral hemorrhage Внутрішньомозковий крововилив

Ischemic stroke Ішемічний інсульт

Abscess Абсцес

Cyst Кіста

Thmor Тмор

155 / 158
A 38-year-old woman, who was diagnosed with systemic lupus erythematosus (SLE) 3 years ago, comes to her physician with a complaint of facial swelling and decreased urination that she first noticed 2 weeks ago. She currently takes azathioprine and corticosteroid. Her vital signs show blood pressure 150/90 mm Hg, pulse – 91/min., temperature – 36.8 C and respiratory rate – 15/min. On physical examination, the doctor notices erythematous rash on her face exhibiting a butterfly pattern. The laboratory studies reveal hypertriglyceridemia and proteinuria. Which of the following is the most likely mechanism of SLE`s complication in this patient? 38-річна жінка, якій 3 роки тому поставили діагноз системний червоний вовчак (СЧВ), звернулася до лікаря зі скаргою на набряк обличчя та зменшення сечовипускання, яке вперше помітила 2 тижні тому.В даний час приймає азатіоприн і кортикостероїд.За життєвими показаннями артеріальний тиск 150/90 мм рт.ст., пульс – 91/хв., температура – 36,8 С і частота дихання – 15/хв.На фіз. при огляді лікар помічає еритематозний висип на обличчі з малюнком метелика Лабораторні дослідження виявляють гіпертригліцеридемію та протеїнурію Що з перерахованого є найбільш імовірним механізмом ускладнення СЧВ у даної пацієнтки?

Acute infection ol-the kidney Гостра інфекція нирок

Decrease in renal blood flow (ischemic nephropathy) Зниження ниркового кровотоку (ішемічна нефропатія)

Increased plasma oncotic pressure Підвищений онкотичний тиск плазми

- -

Immune complex-mediated glomerular disease клубочкова хвороба, опосередкована імунними комплексами

156 / 158
A 20-year-old female comes to the clinic after missing her last 2 periods. Her cycles are usually regular, occurring at 28-30 day interval with moderate bleeding and some abdominal discomfort. She also complains of progressively diminishing peripheral vision. Her doctor reveals loss of vision in the lateral halves of both eyes. Involvement of which of the following structures would you most likely expect to be the reason of bitemporal hemianopsia? 20-річна жінка прийшла в клініку після відсутності останніх 2 менструацій. Її цикли зазвичай регулярні, відбуваються з інтервалом 28-30 днів з помірним кровотеча та певний дискомфорт у животі. Вона також скаржиться на прогресуюче погіршення периферичного зору. Її лікар виявляє втрату зору в бічних половинах обох очей. Ураження якої з наступних структур, на вашу думку, є причиною бітемпоральної геміанопсії?

Left optic tract Лівий зоровий тракт

Left optic nerve Лівий зоровий нерв

Right optic tract Правий зоровий тракт

Optic chiasm Оптичний хіазм

Right optic nerve Правий зоровий нерв

157 / 158
A 24-year-old man undergoes surgery and during the operation, an organ is excised and sent for histological evaluation. A light microscopic examination roved s Ihe organ encased by thin connective tissue capsule that enters the substance oi the lobes to further subdivide the organ into irregular lobular units. Bach lobule contains a cluster of follicles filled with colloid. Follicular epithelium consists of low columnar, cuboidal or squamous cells depending on the level of activity ol the follicle. Which of the following organs does this tissue most likely belong to? 24-річному чоловікові роблять операцію, і під час операції орган вирізають і відправляють на гістологічне дослідження. Світлове мікроскопічне дослідження показало, що орган закритий за допомогою тонкої сполучнотканинної капсули, яка входить у речовину часток для подальшого поділу органу на неправильні часточкові одиниці. Часточка Баха містить скупчення фолікулів, заповнених колоїдом. Фолікулярний епітелій складається з низьких стовпчастих, кубоподібних або плоских клітин залежно від рівня активності. ол фолікула. До якого з наведених органів швидше за все належить ця тканина?

Pancreas Підшлункова залоза

Parathyroid gland Паращитовидна залоза

Parotid gland Привушна залоза

Thyroid gland Щитовидна залоза

Thymus Тимус

158 / 158
A 37-year-old man is admitted to a hospital with mental confusion and disorientation. His wife reports he became more irritable and forgetful in the past year. In addition, she notes that he became a vegan a year ago, and currently, his diet consists of starchy foods like potatoes, corn, and leafy vegetables. Gl symptoms include anorexia, diarrhea and vomiting. He has glossitis and skin lesions that appear as vesicles over the extremities. Eczema-like lesions around the mouth, as well as desquamation and roughened skin over the hands, are also present. Neurologic examination reveals symmetrical hypesthesia for all types of sensation in both upper and lower extremities in a 'gloves and socks' distribution. Deficiency in the diet the of which of the following amino acids is the most likely cause of this condition? 37-річний чоловік потрапив до лікарні зі сплутаністю свідомості та дезорієнтацією. Його дружина повідомляє, що останній рік він став більш дратівливим і забудькуватим. У Крім того, вона зазначає, що він став веганом рік тому, і наразі його дієта складається з крохмалистих продуктів, таких як картопля, кукурудза та листові овочі. Симптоми Gl включають анорексію, діарею та блювання. У нього глосит і ураження шкіри, які виглядають як пухирці на кінцівках.Екземоподібні ураження навколо рота, а також лущення та огрубіння шкіри на руках також присутні.Неврологічний огляд виявляє симетричну гіпестезію для всіх типів відчуття у верхніх і нижніх кінцівках у «рукавичках і шкарпетках» розподіл.Дефіцит якої з наступних амінокислот є найбільш імовірною причиною цього стану?

Arginine Аргінін

Tryptophan Триптофан

Threonine Треонін

Histidine Гистидин

Lysine Лізин